Exámen de Cirugía

December 15, 2016 | Author: Yuliana Mercedes Castellanos Ojeda | Category: N/A
Share Embed Donate


Short Description

Download Exámen de Cirugía...

Description

Examen Módulo de Cirugía

1.- Masculino de 65 años con antecedentes de enfermedad inflamatoria intestinal, que se presenta porque desde hace 6 semanas presenta disminución en el calibre de las heces, hematoquezia y constipación. Se realiza colon por enema encontrando una zona de estenosis en el sigmoides, con imagen en manzana mordida. Estos datos le hacen sospechar fuertemente en: a) b) c) d)

Enfermedad diverticular Cáncer de colon Isquemia Enfermedad de Crohn

Factores de Riesgo: • Dieta alta en grasas. • Edad mayores de 40 años. • Antecedentes Personales de adenomas o carcinoma. • Enfermedad Inflamatoria Intestinal. • Síndromes Familiares. • Poliposis familiar múltiple

Clínica: Un aspecto fundamental en el diagnóstico de las neoplasias de colon y recto es la sintomatología, que puede variar dependiendo de la localización del tumor. Los síntomas más frecuentes son, la alteración del hábito intestinal en forma de estreñimiento o diarrea o disminución en el calibre, la expulsión de sangre o moco, dolor abdominal, masa palpable y síndrome constitucional junto con anemia. En las neoplasias de colon derecho es, en muchas ocasiones, una alteración del estado general junto con anemia y pérdida de peso la única manifestación; Puede acompañarse de dolor abdominal en fosa ilíaca o región subcostal derecha y plenitud postprandial que puede llevarnos a pensar en un cuadro dispéptico. En algunos pacientes se palpa una masa, generalmente en fosa ilíaca derecha.

En los carcinomas de colon transverso y descendente suele existir un estreñimiento progresivo que requiere la administración de laxantes. Otros pacientes presentan diarrea o existe una alternancia estreñimiento-diarrea. Suele existir un peristaltismo aumentado que molesta al enfermo y mejora con la expulsión de gases. En algunas ocasiones las heces se acompañan de sangre y/o moco, siendo esto más evidente en tumores próximos al recto. Tipo 2. El tipo ulcerado es la forma más frecuente de cáncer rectocólico, con bordes prominentes, irregulares y duros al tacto con la pinza de biopsia. Esta ulceración puede crecer y abarcar toda la circunferencia intestinal con un claro compromiso de la luz colónica, cuya imagen radiológica característica es la "estenosis en servilletero" o "manzana mordida". Este tipo es más frecuente en el colon izquierdo, donde suele provocar síntomas de obstrucción intestinal.

Referencias Bibliográficas: 1. Compton, CC. Colorectal Carcinoma: Diagnostic, prognostic and molecular features. Mod Pathol 2003; 16(4): 376-388 2. Compton, CC. Colon and Rectum. Protocol College of American Pathologists. Jannuary 2005. Based on AJCC/UICC TNM, 6th edition. 3. Birbeck, KF et al. Rates of circumferential resection margin involment vary between surgeons and predict outcomes in rectal cancer surgery. Ann Surg 2002; 235 (4): 449457 4. Greene FL et al. A new TNM staging strategy for node-positive (Stage III) Colon Cancer. Ann surg 2002; 236 (4): 416-421 5. Nagtegaal, ID et al. Macroscopic evaluation of rectal cancer resection specimen: Clinical significance of the Pathologist in quality control. J Clin Oncol 2002; 20 (7): 1729-1734.

2.- Masculino de 42 años alcohólico con cirrosis y ascitis. Es hospitalizado por agitación y comportamiento extraño. ¿Cuál de los siguientes hallazgos es el más útil para hacer el diagnóstico de encefalopatía hepática?

a) b) c) d)

Ictericia Asterixis de las manos Hemangiomas superficiales Signo de la ola positivo

ENCEFALOPATÍA • Alteración en el estado mental, en el comportamiento y en el sueño que progresa a la desorientación y al coma. • Indica una insuficiencia hepática severa. • Fisiopatologia

– – – • • •

• •

Las toxinas son inactivadas por el hígado pero entran a la circulación portal por los cortos circuitos. Toxinas no son inactivadas y excretadas. Las toxinas se encuentran elevadas en el líquido cerebroespinal.

TOXINAS. Amonio. – Neuroexcitatorio y depresor del sistema nervioso central. Aminoácidos aromáticos. – Son precursores de neurotransmisores. – Si incrementan en sangre la encefalopatía no progresa. Acido gammaaminobutírico (GABA). – Incrementado significativamente en la encefalopatía. Benzodiacepinas endógenas. – Compiten por los receptores de GABA y barbitúricos.

CUADRO CLINICO El diagnóstico de la encefalopatía hepática depende de la existencia de una enfermedad hepática, desde la insuficiencia hepática aguda y la toxicidad por fármacos, hasta las enfermedades crónicas, tales como la hepatropatía alcohólica o la cirrosis. En la historia clínica se hace énfasis especial en los antecedentes de enfermedades del hígado diagnosticadas con anterioridad, una historia de alcoholismo o de hepatitis. Transfusiones de sangre previas o abuso de drogas por vía intravenosa pueden sugerir una hepatitis crónica B o C. La utilización de fármacos hepatotóxicos, como la metildopa, la nitrofurantoina o la isoniácida, puede ser causa de una hepatopatía crónica, mientras que una dosis alta de paracetamol puede ser causante de una necrosis hepatocelular fulminante. El cuadro clínico incluye tres elementos: cambios en el estado mental, hedor hepático y asterixis. El hedor hepático se refiere al aliento fétido del paciente. La asterixis es un temblor por sacudidas, irregular y bilateral de las manos, debido a una interrupción momentánea y brusca del tono muscular de los antebrazos. El examen físico se centra en la búsqueda de los estigmas característicos de las enfermedades hepáticas: ictericia, nevus en araña, ginecomastia, atrofia testicular, venas distendidas en la pared abdominal (cabeza de medusa) y ascitis. La exploración neurológica durante los estadíos precoces suele mostrar apraxia de construcción y dificultad para escribir. Otros hallazgos físicos son la rigidez de las extremidades e hiperreflexia. En el estadío de coma profundo suele haber pérdida del tono muscular y disminución de los reflejos tendinosos profundos.

RESUMEN: • Petequias. • Rinofima. • Contractura de Dupuytren (aponeurosis palmar). • Telangiectasias en el tronco. • Ascitis.

• • •

Asterixis. Eritema palmar. Atrofia testicular.

Blibliografía Shakelford’s. Surgery of the alimentary tract. 5a. Ed. 2002. Tomo 3. Feldman´s. Gastroeneterology. 2002. Perez. Anatomía y fisiología del hígado. Univ. Católica de Chile. 2005. Bratiz. Serum laboratory test in cirrhosis. Journal of Hepatology. Slovakia. 2005. Paradis. Glycomics. Journal of hepatology. Ireland. Agosto 2005.

3.- Mujer de 33 años de edad. Desde hace 3 días notó fotofobia en ojo derecho y visión borrosa. El cuadro se acompaña de dolor frontal y supraciliar derechos. A la EF se aprecia ojo izquierdo sin datos patológicos. En ojo derecho se observa una pupila miótica y la presión intraocular está disminuída. ¿Diagnóstico más probable?

a) b) c) d)

Uveitis anterior Uveitis posterior Panuveitis Conjuntivitis

‘DEFINICIÓN Uveitis significa inflamación del tracto uveal, que es la capa media del ojo (Figura 1). Puede ser anterior, cuando afecta al iris o a la porción anterior del cuerpo ciliar (recibiendo también el nombre de iritis o iridociclitis). Cuando afecta al vítreo y la pars plana del cuerpo ciliar hablamos de uveitis intermedia, que se conoce como pars planitis cuando presenta un cuadro típico caracterizado por la presencia de grandes acúmulos inflamatorios en la pars plana (“bancos de nieve - snow banks”-) o gruesas condensaciones en vítreo (“bolas de nieve”). La La uveitis posterior es la que afecta a la coroides y en ocasiones por extensión a la retina (coroiditis, coriorretinitis, vasculitis retiniana). Cuando afecta a la totalidad de la úvea hablamos de panuveitis. La parte anterior y la posterior del ojo tienen diferente origen embriológico, por lo que las enfermedades que afectan a una u otra son distintas. La localización anterior es la forma más frecuente y también, junto con la panuveitis, la que con mayor frecuencia se asocia a enfermedades sistémicas. La uveitis anterior (UA) puede diferenciarse clínicamente de la posterior por la aparición de dolor, enrojecimiento ocular y fotofobia (especialmente si el comienzo del cuadro ha sido agudo). Pocas veces existe disminución de la agudeza visual y si aparece suele ser leve por pérdida de transparencia de medios secundaria a la presencia de detritus inflamatorios

y/o edema macular qnístico (EMQ), una afectación de la mácula que frecuentemente acompaña a la UA sin afectación del polo posterior propiamente dicha. A la exploración con lámpara de hendidura se caracteriza por la presencia de células (signo de Tyndall) y aumento de proteínas (“fiare”) en la cámara anterior del ojo. cá

PAC MG-1 B4 PAC MG-1 PROGRAMA DE ACTUALIZACION CONTINUA PARA MEDICOS GENERALES. ACADEMIA NACIONAL DE MEDICINA Director: Dr. Luis Martín Abreu

4.- Niño de 4 años de edad. Al estar jugando con su padre, éste observó un reflejo blanquecino en la pupila del ojo derecho. El paciente no refiere ningún síntoma ni se queja de mala visión. A la EF se encuentra que con el ojo derecho sólo ve “bultos”. El ojo izquierdo posee buena visión y características normales. El examen del fondo de ojo derecho reveló una más blanquecina de bordes difusos y corpúsculos de color blanco en la cavidad vítrea. ¿Cuál es el diagnóstico más probable?

a) b) c) d)

Retinoblastoma Retinopatía del prematuro Toxocariasis ocular Carcinoma intraepitelial

RETINOBLASTOMA DEFINICIÓN Y EPIDEMIOLOGÍA Tumor congénito de retina altamente maligno. Incidencia varía 1:17,000 a 1:34,000 nacidos vivos. Edad promedio de diagnóstico: 2 años. MANIFESTACIONES CLÍNICAS l Leucocoria. (pupila blanca) l Estrabismo. l Ojo rojo, doloroso. l Pobre agudeza visual. METÁSTASIS l Principalmente por vía hematógena a diversos tejidos (pulmón poco frecuente). l Extensión a nervio óptico y sistema nervioso central. l Finalmente por vía linfática después su extensión a la órbita. TRATAMIENTO Irradiación externa, enucleación y en los casos con extensión a órbita o metástasis se usa quimioterapia (ciclofosfamida, doxorrubicina y vincristina).

Leucocoria. Diagnóstico diferencial del retinoblastoma Dr. A. de las Heras - Dr. J. Abelairas - Dr. J. Peralta - Dr. J.L. Encinas

5.- Paciente de 72 años de edad con antecedente de Infarto cardiaco hace 6 meses se presenta a sala de urgencias por dolor intenso de muslo y pierna izquierda, palidez, e hipotermia. A la exploración física se encuentra FC de 136x’ arrítmica, ausencia de pulsos poplíteo y distales, con rigidez del pie y la pierna con piel marmórea. El tratamiento indicado es:

a) b) c) d)

colocación de injerto femoropoplíteo distal estabilización y amputación supracondílea estabilización e inicio con acenocumarina 3 mg c/24 hrs estabilización y angioplastía percutanea



Es la interrupción súbita del flujo arterial con el desarrollo de hipoperfusión e isquemia distal del órgano afectado.



Las principales causas son:

   

Embolismo 80%, trombosis10%, traumatismo 5%, iatrógeno 5%

Embolismo de origen cardiaco: 80% de todos los casos FIBRILACION AURICULAR IAM VALVULOPATIA REUMATICA

Manifestaciones clínicas 1. Dolor (Pain) 2. Parestesias 3. Parálisis 4. Palidez 5. Poiquilotermia 6. Pulsos periféricos ausentes

Clasificación de Rutherford (SVS) 1, 2ª, 2B Y



3: Extremidad No viable:  Parálisis, hipotermia severa, anestesia sin señal doppler ni arterial ni venosa, piel marmórea.  Tratamiento: amputación

BIBLIOGRAFIA: Vascular Surgery Rutherford  6th edition, vol 1 pp 959-1000  2006 Elsevier, USA

6.- Paciente masculino de 45 años de edad postoperado de interposición de injerto femoro femoral por lesión por arma punzocortante en el muslo derecho, presenta edema importante desde la rodilla hacia abajo, parestesias y llenado capilar retardado pero temperatura normal. El diagnóstico mas probable en el que hay que pensar es: a) trombosis venosa profunda b) trombosis del injerto

c) síndrome compartamental d) infección de la extremidad.

CONCEPTO Podemos definir el Síndrome Compartimental Agudo como el conjunto de signos y síntomas secundarios al aumento de la presión en una celda fascial de un miembro, lo cual provoca una disminución de la presión de perfusión capilar comprometiendo la viabilidad de los tejidos de dicha celda. No debemos confundir el SCA con la Contractura Isquémica de Volkman. La segunda es una secuela de la primera. Podemos definir la Contractura Isquémica de Volkman (CIV) como el conjunto de secuelas morfológicas y funcionales de la necrosis muscular y nerviosa que sigue a un SCA no tratado correctamente o de mala evolución. Las características de la CIV son: - Alteraciones neurológicas. - Alteraciones cutáneas. - Alteraciones articulares. - Alteraciones musculares. El Síndrome Compartimental Crónico es el aumento transitorio de la presión intracompartimental como consecuencia de movimientos repetidos o ejercicios físicos. Se da fundamentalmente en el miembro inferior y es una enfermedad crónica. Se caracteriza por dolores tipo calambre que aparecen durante el ejercicio físico y ceden con el reposo. ETIOLOGIA Las causas del SCA podemos englobarlas en dos grandes grupos: III.1 Las que provocan una disminución del compartimento: - Vendaje o yeso compresivo. - Quemaduras y congelaciones: se producen unas escaras duras, que no son elásticas que pueden llegar a ocasionar un SCA. Para evitarlo se debe proceder a quitar las escaras. - Cierre incorrecto de celdas aponeuróticas, es decir con excesiva tensión. - Aplastamiento. III.2. Las que provocan un aumento del contenido del compartimento: - Edema postisquemia (lesión arterial, tromboembolismo arterial, cateterismo arterial...) - Hematoma primitivo (hemofilia, tratamiento anticoagulante...) - Hemorragias intracompartimental (fracturas, osteotomías...) - Envenenamiento por mordedura. CLINICA Dolor que aumenta con la extensión pasiva Tensión Hinchazón Alteraciones sensitivas Disminución de la motilidad Cianosis Disminución del pulso arteria principal

BIBLIOGRAFIA 1. Botte, MJ; Keenan, MA; Gelberman RH: Volkmann´s ischemic contracture of upper extremity. Hand Clin, 1998: 14 (3): 483-97. 2. Botte,MJ; Fronek, J; Pedowitz, RA: Exertional compartment syndrome of the upper extremity. Hand Clin, 1998: 14 (3): 477-82. 3. Van Essen, GJ; McQueen, MM: Compartment syndrome in the lower limb. Hosp Med, 1998 59(4): 294-7. 4. DBotte, MJ; Gelberman, RH: Acute compartment syndrome of the forearm. Hand Clin, 1998: 14 (3): 391-403. 5. Gellman, H; Buch, K: Acute compartment syndrome of the arm. Hand Clin, 1998: 14 (3): 385-9. 6. Hovius, SE; Ultee, J: Volkmann´sischemic contracture. Prevention and treatment. Hand Clin, 2000: 16 (4): 647-57.

7.- Masculino de 24 años acude al servicio de medicina externa refiriendo dolor faríngeo importante, odinofagia, otalgia, disfagia y fiebre, a la EF. Se encuentra con 39.1 grados de temperatura, ronco, con tendencia la somnolencia, el examen de la faringe muestra una amígdala derecha hipertrófica que se desplaza hacia abajo y hacia el centro, existe trismos y adenomegalias cervicales, cual de los siguientes es el diagnostico más probable:

a) b) c) d)

Faringitis exhudativa. Cáncer de amígdalas. Absceso periamigdalino. Mononucleosis infecciosa.

El absceso periamigdalino (apa) es la infección más frecuente de los tejidos profundos de cabeza y cuello y la complicación más usual de la faringoamigdalitis aguda. un 10-20% de los pacientes tienen antecedentes de amigdalitis frecuentes. Ocasionalmente ocurre en pacientes amigdalectomizados. Se produce por invasión bacteriana del espacio periamigdalino (entre la cápsula amigdalar y Sos músculos constrictor superior de la faringe y palatofaríngeo). Inicialmente es un flemón, con posibilidad posterior de formación de una colección purulenta (absceso). Generalmente unilateral (3% bilateral). la mayoría se localizan en el polo superior de la amígdala, dada la localización del proceso requiere un diagnóstico y tratamiento precoces, ya que puede producir un compromiso respiratorio por compresión o por drenaje espontáneo y broncoaspiración. también puede extenderse a otras zonas por proximidad: espacio parafaríngeo, prevertebral y mediastino. la recurrencia del absceso se produce en 10-15% de los pacientes, 4 veces más frecuente en niños con antecedentes de amigdalitis de repetición. se ha descrito un aumento de la incidencia de esta infección en la edad pediátrica en los últimos años. Manifestaciones clínicas: tras una faringoamigdalitis aguda, con o sin tratamiento antibiótico, aparece fiebre, odinofagia acusada e irradiada al oído, disfagia más o menos intensa (babeo), voz dificultosa, apagada, halitosis bucal, adenopatía dolorosa cervical homolateral y trismo en mayor o menor grado, que puede dificultar la

exploración local: abultamiento asimétrico de la amígdala con desplazamiento de la misma hacia la línea media y abajo, y de la úvula hacia el lado opuesto. El diagnóstico es clínico. Debe diferenciarse el flemón del absceso, siendo a veces necesaria ecografía o tc, pero la obtención de pus por punción o escisión es más rápida y económica. La escasa colaboración de los pacientes pediátricos dificulta el diagnóstico y el tratamiento. La infección está causada la mayoría de las ocasiones por una flora mixta (aerobios y anaerobios). El germen más frecuente es el streptococcus pyogenes. El antibiótico de elección es la amoxicilina con ácido clavulánico. En alérgicos a la penicilina se recomienda el uso de la clindamicina. Generalmente es necesario el ingreso hospitalario para tratamiento antibiótico parenteral y para valorar la formación de un absceso que deba ser evacuado. La mayoría de los pacientes responde bien al tratamiento médico. Algunos pueden ser tratados de forma ambulatoria tras la evaluación y tratamiento inicial.

TABLA I. Clínica del absceso periamigdalino • Fiebre de cualquier grado. • Dolor de garganta acusado y localizado pudiendo irradiarse al oído. • Disfagia asociada o no a babeo. • Dificultad para hablar con una voz característica apagada, “papa-caliente”. • Olor fétido en la boca al respirar. • Trismo de intensidad variable. Distancia interdental: – 2,5-4 cm (grado I) – 1-2,5 cm (grado II) – < 1 cm (grado III) – imposibilidad de mover la mandíbula (total) • Adenopatía dolorosa cervical del mismo lado. • El paciente llega a estar temeroso, aprensivo y pálido. • Exploración local: inflamación lateral y superior a la amígdala con desplazamiento medial y anterior de la misma y una úvula edematosa dirigida hacia el lado opuesto. La amígdala está eritematosa, inflamada y con exudado blanquecino aunque a veces está oculta por el tejido inflamado adyacente. Paladar blando del lado afecto edematoso y enrojecido. • Laringoscopia indirecta: edema supraglótico y parafaríngeo.

Absceso periamigdalino Fernando Álvez González Raut VV, Yung MW. Peritonsillar abscess: the rational for interval tonsillectomy. Ear Nose Throat J 2000,79: 206-209. Suskind DL, Park J, Picirillo JF et al. Conscious sedation. A new approach for peritonsillar abscess drainage in the pediatric population. Arch Otolaryngol Head Neck Surg 1999, 125: 1197-1200. Yellon RF. Infections of the fascial spaces of the head and neck in children. Semin Pediatr Infect Dis 1998, 9:60-69.

8.- Femenino de 55 años que acude al servicio por presentar pérdida ponderal moderada, calambres abdominales y diarrea intermitente no sanguinolenta. A la EF el abdomen está distendido y hay una masa palpable en FD y FID; la TAC demuestra una masa inflamatoria

en éstas zonas con engrosamiento del íleon terminal y el colon proximal. El diagnóstico más probable es:

a) b) c) d)

CUCI Enfermedad de Crohn Colon irritable Intolerancia a la lactosa

DEFINICIÓN:  Inflamación crónica que afecta a cualquier porción del tubo digestivo.  La lesión más temprana es la úlcera superficial ubicada sobre un folículo linfoide.  Es transmural y panintestinal, a menudo puede resultar en la formación de fístulas.

 ETIOLOGÍA.  Agentes infecciosos: Mycobacterium paratuberculosis, Listeria monocytogenes. Paramixovirus. 

Defectos en la permeabilidad de la mucosa

 PATOGENIA. Evento inicial:  Infecciones  Toxinas  Antiinflamatorios  Perpetuación del evento:  Bacterias luminales  Productos bacterianos  Antígenos de la dieta CUADRO CLINICO. 



SÍNTOMAS

CROHN.

DIARREA

+++

HEMORRAGIA RECTAL

+

TENESMO

0

DOLOR ABDOMINAL +++ FIEBRE

++

VÓMITOS

+++

PÉRDIDA DE PESO

+++

SIGNOS

CROHN.

AFECCIÓN +++ PERIANAL MASA ABDOMINAL +++ MALNUTRICIÓN

+++







DIAGNÓSTICO.  LABORATORIO  BIOMETRIA HEMATICA.  PRUEBAS DE FUNCIONAMIENTO HEPÁTICO.  PROTEÍNA C REACTIVA  VSG  PRUEBA DE SHILLING  HLA A2 DIAGNÓSTICO.  TOMOGRAFÍA.  Abscesos  Fístulas  Estenosis  Adenopatías  Complicaciones  Perianales  Paraestomales DIAGNÓSTICO.  Ultrasonido  Anormalidad de la pared  Abscesos  Fístulas  Ultrasonido doppler  Cambios en el flujo de la mesentérica indica actividad de la enfermedad

CARACTERISTICAS MACROSCÓPICAS ENFERMEDAD DE CROHN. Extensión Discontinua Engrosamiento Marcado Compromiso de la luz Marcado Estenosis Frecuentes cicatriciales Mucosa residual Edematosa Seudopólipos Raros Serosa Afectada Ganglios linfáticos Agrandados Ileón terminal Afectado Afectación rectal En el 50-75% Fistulización Frecuente Ulceraciones Delimitadas Lesiones anales Frecuente

Blibliografía Feldman´s. Gastroeneterology. 2002.

9.- Un hombre de 35 años acude a consulta porque tiene dolor sordo perineal y persistente, disuria de 6 meses de evolución. Niega IVUS y descarga uretral. Su temperatura es de 37°C . Al examen rectal la próstata es ligeramente dolorosa, pero no esta aumentada de tamaño ni indurada. Uroanálisis normal. Secreción prostática, muestra 30 leucos por campo sin bacterias. Cultivos de secreción prostática y orina negativos. Cuál es el diagnóstico más probable.

a) b) c) d)

cistitis aguda prostatitis aguda Prostatitis bacteriana crónica Prostatitis no bacteriana crónica

La Prostatitis es uno de los más comunes problemas urológicos. Cerca del 50% de los hombres experimentan un episodio de prostatitis una vez en la vida. Debido a que la Hiperplasia Prostática Benigna (HPB), el Cáncer de Próstata (CP) y la Prostatitis Crónica Abacteriana (PCA) pueden coexistir, es difícil distinguir entre estas entidades. Los síntomas de PCA y de HPB se superponen de tal forma que muchos ancianos reciben equivocadamente éste último diagnóstico. Aunque a diferencia de la HPB y el CP que son preferentemente del hombre anciano, la PCA puede presentarse a cualquier edad. La Prostatitis puede elevar los niveles de Antígeno Prostático Específico (PSA) lo que conduce a la realización de un gran número de biopsias prostáticas innecesarias. En la Prostatitis Crónica se desconoce no sólo qué la causa sino cuál es la mejor forma de manejarla. Drach y col clasificaron y definieron a la Prostatitis de la siguiente forma: * Prostatitis Bacteriana Aguda: se define por la recuperación de bacterias del fluido prostático, fluido purulento y síntomas sistémicos de infección (fiebre, mialgia , etc.) * Prostatitis Bacteriana Crónica: recuperación de bacterias en número significativo del fluido prostático en ausencia de infección urinaria o signos significativos de infección sistémica. * Prostatitis Abacteriana: sin recuperación de número significativo de bacterias del fluido prostático pero con pus microscópica en el mismo * Prostatodinia: sin recuperación de bacterias o pus en el líquido prostático, pero el paciente tiene urgencia urinaria persistente, disuria, discomfort prostático. El National Institute of Health (NIH) de USA estableció en 1995 un nuevo sistema de clasificación. Es el siguiente: * Prostatitis Tipo I: infección aguda de la próstata. Corresponde a la Prostatitis bacteriana aguda de la antigua clasificación. * Tipo II: infección recurrente de la próstata. Corresponde a la Prostatitis Bacteriana Crónica * Tipo III: Sin infección demostrable. Corresponde a Prostatitis Crónica Abacteriana/Sindrome de Dolor pélvico crónico. Tipo III A: leucocitos en semen. Tipo III B: Sin leucocitos en semen. * Tipo IV: sin síntomas subjetivos, detectado por biopsia prostática o por la presencia de

leucocitos en secreciones prostáticas. Corresponde a la Prostatitis inflamatoria asintomática. La etiología de la Prostatitis es desconocida en el 90% de los casos, siendo bacteriana en el 10% de los casos. La literatura antigua describe como determinantes potenciales de esta condición los niveles de hormonas sexuales, dieta, enfermedades del tracto urinario, stress, factores psicológicos, alergia. Estudios más recientes examinaron la edad, etnicidad, agentes infecciosos, niveles de ácido úrico, actividad sexual, cálculos y quistes prostáticos, citoquinas proinflamatorias y biopsia prostática. Bibliografía 1- McNaughton C. Diagnosis and Treatment of Chronic Abacterial Prostatitis: a Systematic review. Ann Intern Med 2000 ; 133 : 367-381 2- Leskinen M , Lukkarinen O, Marttila T. Effects of finasteride in patients with inflammatory chronic pelvic pain syndrome: a double blind, placebo-controlled, pilot study. Urology 1999 ; 53 : 502-505

10.- Un hombre de 45 años acude a urgencias debido a un severo dolor en flanco derecho que inció abruptamente hace 3 horas. El dolor es intermitente de tipo cólico y se irradia hacia el testículo ipsilateral. Refiere nauseas y se encuentra en constante movimiento sobre su cama. Temperatura de 37°C, el dipstick demuestra hematuria, pH 5.8. Cuál es el siguiente paso en el diagnóstico.

a) b) c) d)

Pielografía IV Rx de abdomen Electrolítos séricos con Ca Urocultivo

DOLOR TESTICULAR AGUDO Es el cuadro doloroso que tiene una duración inferior a 7 días. La epididimitis se considera una excepción puesto que por definición, un cuadro agudo es menor a 6 semanas de evolución.5

Causas Extratesticulares: hernia inguinoescrotal, funiculitis, edema escrotal idiopático, gangrena de Fournier, obstrucción ureteral aguda por cálculos ureterales pelvianos. Causas testiculares: torsión del cordón espermático, torsión de los apéndices testiculares, orquiepididimitis, traumatismos testiculares, complicaciones de hidroceles, neoplasias del testículo, infarto testicular idiopático, trombosis de la vena espermática entre otras.

Litiasis ureteral: Por el tipo de inervación del uréter, el cólico nefrítico se irradia al testículo ipsilateral. RECOMENDACIONES PARA EL DIAGNOSTICO DE LA LITIASIS RENAL Y URETERAL 1. Se debe tener un alto índice de sospecha de cálculo reno-ureteral ante la presencia de dolor cólico severo, de aparición súbita, localizado en el ángulo costovertebral e irradiado al flanco, hipogastrio, hemiescroto o labios mayores, acompañado o no de síntomas urinarios irritativos y que pueden estar asociados a náuseas y vómito. 2. La presencia de hematuria, definida como más de tres hematíes por campo de alto poder, debe llevar a una investigación imagenológica del trato urinario, en busca de la etiología. Hay mayor posibilidad de que esa hematuria sea secundaria a un cálculo en los menores de 50 años, pues en los mayores hay más probabilidad de otros diagnósticos, que también son muy importantes (tumores renales y de vejiga, H.P.B., etc.) 3. Ante una infección urinaria persistente, la evaluación urológica se debe realizar independientemente del germen aislado. Es alta la relación entre microorganismos ureasa positivos y cálculos coraliformes; sin embargo, la mayoría de los cálculos de estruvita, que también son infecciosos, no son coraliformes y se pueden asociar a microorganismos ureasa negativos. No es prerrequisito la presencia de gérmenes ureasa positiva para la formación de cálculos de estruvita. De otra parte, la mayoría de las veces los cálculos de oxalato de calcio no se asocian con infección 4. Es importante la implementación de laboratorios con capacidad para el análisis de los cálculos, puesto que su composición tiene implicaciones con posibilidades de recidiva y de tratamiento preventivo. 5. La posibilidad de que un cálculo asintomático se torne sintomático aumenta el número de antecedentes clínicos previos de litiasis renal y con el número de cálculos encontrados en el momento del diagnóstico incidental. 6. Es importante el seguimiento de los pacientes con antecedentes de cálculos infecciosos, episodios repetidos de litiasis, antecedentes de intervenciones terapéuticas y anormalidades anatómicas. 7. La litiasis en niños, por estar asociada con infección urinaria y alteraciones anatómicas, requiere seguimiento estricto. Considerando la historia familiar y las alteraciones metabólicas, como hipercalciuria, acidosis tubular renal o cistinuria, que con frecuencia se encuentra, es obligatorio el estudio metabólico. 8. En la mayoría de los pacientes, la urografía es el examen de elección para el diagnóstico de la litiasis renal y ureteral, pues no sólo permite la identificación y localización de los cálculos, sino que además nos permite evaluar el grado de dilatación y el estado del riñón contralateral, dando una idea de la función renal. Por otra parte, es un examen que se puede llevar a cabo en cualquier ciudad del país y con el que están familiarizados los técnicos de radiología y muchos médicos generales.

9. La ecografía renal más la radiográfica simple de abdomen es una buena alternativa en los casos en los cuales exista contraindicación para la urografía. Sus limitaciones son: no permitir la evaluación objetiva del grado de obstrucción, requerir de un nivel «adecuado» de hidratación para determinar la dilatación del sistema colector (falsos positivos) y que su realización e interpretación son operador-dependiente. Se considera, entonces, que su mejor aplicación es para el seguimiento de cálculos ya diagnosticados.

BIBLIOGRAFIA 1. Resnick M., Caldamone A. and Spirnak P.: FLANK PAIN In: DECISION MAKING IN UROLOGY. Philadelphia: B.C. Decker. P. 22. 1991. 2. Gearhart J.P. and Jeffs R.D.: CHILDHOOD UROLITHIASIS: EXPERIENCES AND ADVANCES. (Abstract) Pediatrics, 87:445, 1991. 3. Boridy I., Maklad N. and Sandler C.: SUSPECTED UROLITHIASIS IN PREGNANT WOMEN: IMAGING, ALGORITHM AND LITERATURE REVIEW. AJR, 167:869, 1996. 4. Shaw S. T., Poon S. Y. and Wong E. T. : ROUTINE URIANALYSIS: IS THE DIPSTICK ENOUGH?. JAMA 253 (11)»1596, 1985. 5. Messing E., Young T., Hunt V., Newton M., Bram L. and Wegenke J.: HEMATURIA HOME SCREENING: REPEAT TESTING RESULTS. J. Urol. 154,57, 1995. 6. Britton J.P., Dowell A., Whelan P. and Harris C.: A COMMUNITY STUDY OF BLADDER CANCER SCREENING BY THE DETECTION OF OCCULT URINARY BLEEDING. J. Urol. 148:788, 1992. 7. Messing E., Young T., Hunt V. and Wehbie J.: THE SIGNFICANCE OF ASYMPTOMATIC MICROHEMATURIA IN MEN 50 OR MORE YEARS OLD: FINDINS OF A HOME SCREENING STUDY USING URINARY DIPSTICKS. J. Urol. 137:919,

11.- Se trata de paciente femenino de 30 años de edad la cual sufre caída sobre su hombro izquierdo al conducir su bicicleta en una competencia. En la exploración presenta deformidad y signo de la “tecla de piano” en la extremidad distal de la clavícula. ¿Cuál es el diagnóstico más probable de de la lesión?:

a) Luxación acromio-clavicular. b) Fractura de la cabeza humeral. c) Luxación escápulo-humeral. d) Fractura de escápula.

La luxación acromio-clavicular es una de las lesiones traumáticas del hombro que día a día se hace más común, en virtud al aumento de la frecuencia de accidentes de tránsito y deportivos, evidente en la última década. El aporte funcional de la articulación acromio clavicular a la biomecánica del hombro, fuera de contribuir con el 20% del total de los arcos de movimiento, y quizás más importante, es

el de actuar como fulcro, sobre el cual se completan la abducción y flexión del hombro, importantísimas para el desempeño armónico y funcional de la extremidad superior.

La mayoría de los pacientes sufre la lesión en actividades deportivas y accidentes automovilísticos, y dependiendo de la duración e intensidad de las fuerzas aplicadas, se producirán las lesiones descritas como G I, G II y G III de Allman1: • G I: Lesión intraarticular sin ruptura del complejo capsuloligamentoso acromioclavicular, generalmente producida por impacto directo de mediana intensidad sobre la cara externa del acromión. • G II: Subluxación de la clavícula por ruptura de la cápsula y de los ligamentos acromioclaviculares, por una fuerza aplicada en dirección superior y lateral, produciendo inicialmente una rotación externa de la escápula que toma como punto de pivote los ligamentos coracoclavículares. • G III: Luxación completa de la clavícula en dirección postero superior por ruptura de los ligamentos coracoclaviculares, debido a la progresión de la fuerza lesionante descrita en el G II. Generalmente se encuentran desgarros de los músculos deltoides y trapecio en sus inserciones a nivel de clavícula distal y acromión.

DIAGNÓSTICO CLÍNICO A. Mecanismo de trauma B. Cuadro clínico II. Signos de trauma agudo (escoriación equinosis etc. aspecto postero lateral del hombro). II. Dolor III. Limitación funcional IV. Deformidad (signo de la tecla y signo de la charretera)

DIAGNÓSTICO RADIOLÓGICO Se estableció como técnica estándar en este estudio, de acuerdo a lo indicado en la literatura habitualmente (L. Böhler 1957 citado por otros, 4), la toma de placas AP de la articulación acromio-clavicular (es necesario especificarlo así, pues rutinariamente en las placas simples de hombro, el tiempo de exposición mayor hace que la articulación acromioclavicular se vea radiolúcida, (Zariczny

1. Weaver, J.K., Dunn, H.: Treatment of acromioclavicular injuries, especially complete acromioclavicular separation. J. Bone and Joint Surg., 54-A; 1.187-1.194, 12.- Ingresa al servicio de Traumatología masculino de 35 años postraumatizado por colisión automovilística con cuadro de pérdida progresiva de fuerza en miembros inferiores, los reflejos osteotendinosos están abolidos. Al realizarse radiografías simples muestran una fractura por compresión de L1 con desplazamiento del muro posterior y acuñamiento anterior de un 50%. ¿Para valorar la ocupación del canal raquídeo, cual de las siguientes pruebas indicaría?

a) Una gammagrafía ósea. b) Una tomografía cervical. c) Rx ap y lateral con foco en L5 d) Una TAC vertebral centrada en región dorso lumbar. . Tomografía computarizada (TC) En líneas generales, podemos decir que sus indicaciones (con carácter urgente) en los TRM, son todas aquellas lesiones detectadas o sospechadas en las radiografías simples, que puedan suponer riesgo de lesión medular por desplazamientos ulteriores (lesiones inestables), o que ya estén produciendo daño neurológico susceptible de mejorar o estabilizarse tras descompresión quirúrgica. Por lo tanto debe realizarse en todos los pacientes con fracturas, luxaciones y fracturas-luxaciones inestables, y en aquellos con déficit neurológico, preferentemente incompleto. En pacientes con lesiones estables en las radiografías simples y sin déficit neurológico, puede diferirse en función de la presión asistencial y de la disponibilidad del servicio de Radiología. En la práctica, suele indicarse también cuando no se visualizan determinadas zonas de la columna, generalmente C1-C2 y C6-Dl. En pacientes con TCE grave deben realizarse cortes de estas zonas, si no se ven claramente en las radiografías o en el "scout" cervical, e incluso de forma rutinaria, dada la frecuente asociación de estas lesiones. La TC proporciona una excelente visualización de las estructuras raquídeas principalmente de los elementos posteriores y del canal medular, por lo que se pueden ver con nitidez los desplazamientos y fragmentos óseos que puedan estrecharlo o invadirlo. Asimismo, pueden verse fracturas o desplazamientos inadvertidos en las radiografías y nos permite valorar mejor estas lesiones, definiendo claramente las líneas de fractura, cuantificando exactamente los desplazamientos, y en definitiva, evaluando la estabilidad vertebral. Habitualmente se realizan cortes de 5 mm de espesor, aunque la exploración detallada de la columna cervical pueda requerir cortes más delgados, lo que alargará el tiempo de estudio, factor importante en el manejo de los traumatismos. Los equipos de últimas generaciones han conseguido acortar este tiempo y permiten además, la reconstrucción sagital o coronal a partir de los cortes axiales, mejorando la definición y la evaluación del canal medular. La TC es útil también para valorar las articulaciones interapofisarias y los agujeros de conjunción, así como los hematomas paravertebrales y retroperitoneales. Una ventaja adicional es la de ofrecernos información suplementaria sobre ciertas partes blandas del cuello y de las cavidades torácica y abdominal. Las fracturas horizontales que no coincidan con el plano de la TC pueden no visualizarse, como la de odontoides o algunas por compresión. Los hematomas epidurales, hematomielia y hernias discales pueden verse también con TC, aunque la RM define mejor estas lesiones.

La contusión y el edema medular, las lesiones y avulsiones radiculares y los desgarros durales requieren estudio mielográfico adicional y/o RM. La mielo-TC se realiza con inyección de contraste intratecal por punción lumbar o cervical, que obliga a la movilización del paciente o retirada del collarín cervical, además de otros inconvenientes como tiempo de estudio y reacciones adversas, por lo que generalmente no son útiles en el manejo urgente del paciente con TRM.

Balliger, Phillip W. Cerril. Atlas de posiciones radiográficas y procedimientos radiológicos. 7.ª ed.; España: Masson. – Goaz P. W. Radiología oral (principios e interpretación). 3.ª Ed. España; ed.; Mosby.1995

13.- Un varón de 65 años lee en el periódico que el antígeno prostático específico es una buena prueba de investigación para el cáncer y pide a su internista que se la haga. La prueba revela un aumento de antígeno prostático específico de 10.4 ng/ml. El tacto rectal revela una próstata de tamaño normal, pero en la ecografía se encuentra un área hipoecoica pequeña que mide 5x7 mm en el lóbulo derecho. ¿Cuál de las siguientes medidas es el siguiente paso apropiado? Repetir el análisis de antígeno prostático específico en tres meses para verificar si hay aumento ulterior b) Practicar biopsia transrectal del área anormal encontrada en la ecografía c) Comenzar el tratamiento con leuprolida de depósito d) Efectuar una tomografía computadorizada del retroperitoneo, la pelvis y la próstata

a)

Allen R. M. MMS Medicina Interna. 5ª. Edición. National Medical Series. Mc. Graw Hill. 2006. (capítulo 4 X C 4 a (3), b (1)). El antígeno prostático específico es una prueba que puede usarse para detección de cáncer de próstata. Sin embargo, los valores de éste también pueden estar un poco altos en la hipertrofia prostática benigna. Una ecografía transrectal puede identificar lesiones pequeñas no palpables en la exploración rectal. Si los pacientes tienen PSA alto y se confirmó en la ecografía un área anormal, se puede practicar biopsia por vía transrectal bajo guía ecográfica. En general, a los varones con cáncer de próstata se les estudia en busca de metástasis con gammagrama óseo; radiografía de tórax; tomografía computarizada de retroperitoneo y pelvis; o resonancia magnética de retroperitoneo y pelvis con énfasis en la próstata, acompañados por estudios de laboratorio. El carcinoma prostático metastásico puede tratarse con leuprolide, un agonista de la hormona liberadora de hormona luteinizante que suprime la producción testicular de testosterona. El tratamiento con leuprolide equivale a orquiectomía o a terapéutica con estrógenos en cáncer prostático o metastásico.

14.- Se trata de paciente femenino de 50 años de edad que acude a consulta refiriendo que desde hace varios meses presenta dificultad para tragar tanto líquidos como sólidos, así como regurgitaciones de comida sin digerir. Se determina realizar una manometría esofágica la reporta los siguientes hallazgos: ausencia de peristaltismo en el cuerpo

esofágico, hipertonía y una relajación incompleta del esfínter esofágico inferior tras la deglución, con los resultados anteriores el diagnóstico más probable es:

a) Peristalsis esofágica sintomática. b) Esofagitis por reflujo. c) Acalasia. d) Esclerodermia.

La acalasia consiste en la incapacidad para relajar las fibras de músculo liso del aparato gastrointestinal en cualquier sitio de unión de una parte con otra. La acalasia esofágica, o la incapacidad del esfínter gastroesofágico para relajarse al deglutir, por degeneración de las células ganglionares en la pared del órgano.1 El esófago torácico también pierde la actividad peristáltica normal y se vuelve dilatada produciendo un megaesófago. La acalasia esofágica o simplemente acalasia es una rara enfermedad en la cual el esófago se encuentra inhabilitado para llevar el alimento hacia el estómago. La enfermedad afecta ambos sexos y puede aparecer a cualquier edad, sin embargo se diagnostica generalmente entre la tercera y la cuarta década de la vida. Los síntomas más notables son: 1. Dolor retroesternal, que en fases iniciales es intermitente y que se va haciendo progresivo. 2. Disfagia esofágica (el alimento, una vez tragado, se "atasca" por el aumento de presión de la parte distal del esófago y el cardias). 3. En fases avanzadas, se puede dar regurgitación, dolor torácico y pérdida de peso que puede confundir con un cáncer de esófago. Disfagia que se inicia de forma brusca, generalmente a los líquidos, casi siempre relacionada con grandes emociones, de evolución caprichosa, con períodos de deglución normal y que se acompaña en ocasiones de dolor retroesternal por espasmos, y regurgitaciones de líquido claro e insípido, la cual puede ser en la noche y provocar síntomas respiratorios. La presencia de sialorrea llama la atención, así como la halitosis, casi siempre presente. En la acalasia tipo I el dolor es raro y el paciente regurgita por rebosamiento, casi siempre cuando está dormido. En la acalasia tipo II predomina el dolor al deglutir o de forma espontánea y la regurgitación es inmediata.

Gisbert J.P., Losa C., Barreiro A., Pajares J.M. Servicio de Aparato Digestivo, Hospital de la Princesa, Universidad Autonoma de Madrid. Rev Clin Esp 2000 Aug; 200(8):424-31

15.- Masculino de 18 años que es enviado al servicio de otorrino por presentar una tumoración en la línea media del cuello, la cual es renitente, no dolorosa y asciende al protruir la lengua. El diagnóstico clínico más probable es: a) Quiste Branquial b) Quiste demoide

c) Qiste tirogloso d) Linfangioma cervical. Los quistes del conducto tirogloso constituyen las lesiones cervicales congénitas más frecuentes en los niños. Habitualmente son detectados entre los 2 y 10 años de edad aunque un porcentaje importante no son identificados sino hasta después de los 20 años. No existe una predilección por sexo. La forma de presentación clínica clásica es la de una lesión redondeada, lisa, blanda, indolora, ubicada en la línea media y en relación al hueso hioides. Existe un pequeño porcentaje de presentaciones sublinguales o supraesternales. Como el quiste tiene fijaciones al hueso hioides y al foramen ciego de la lengua, puede ser traccionado hacia arriba, cuando el niño traga o saca la lengua. El diagnóstico es confirmado por ultrasonido, que corrobora la lesión quística. Este estudio habitualmente permite también evaluar la presencia de la glándula tiroides. Al no encontrarse ésta en el examen, es preciso realizar una cintigrafía, ya que en esos casos el único tejido tiroideo existente pudiera estar en relación al conducto o quiste tirogloso. Embriológicamente estas lesiones son consecuencia de la falla de obliteración del conducto tirogloso posterior al descenso de la glándula tiroides alrededor de la sexta semana de vida fetal. Anatomo-patológicamente los quistes se encuentran revestidos de epitelio pseudeoestratificado columnar o estratificado escamoso y glándulas secretoras de mucus. La complicación habitual de estas malformaciones es la infección con la flora bacteriana de la boca, consecuencia de su comunicación persistente con la base de la lengua a través del foramen ciego. Los quistes infectados presentan signos inflamatorios y pueden drenar al exterior. El tratamiento recomendable es la extirpación quirúrgica y así prevenir su infección. La técnica fue descrita en 1920 por Sistrunk e incluye la extracción del centro del hueso hioides, por cuyo interior va el conducto tirogloso, y así, evitar la recidiva. La extirpación debe llegar hasta el foramen ciego lingual. En presencia de infección quística, es necesario tratar ésta con antibióticos y posteriormente operar.

16.- Paciente masculino de 60 años de edad, con obesidad exógena manifestando cuadro doloroso de 3 días de evolución, súbito, en hipocondrio derecho, irradiado a región escapular derecha, acompañado de nausea, vómito amargo y de color amarillento, agregándose a las 24 hs del inicio, ictericia de escleras y tegumentos, acolia y coluria. Como antecedente, refiere cuadros dolorosos similares desencadenados por la ingesta de alimentos grasos. ¿Cuál el su diagnóstico mas probable? a) b) c) d)

Hepatitis viral. Colangitis. Coledocolitiasis. Pancreatitis.

La forma clásica de presentación de la coledocolitiasis es la coexistencia de dolor, ictericia y fiebre. Sin embargo, esta tríada sólo se observa en un tercio de los pacientes; en la mayoría, la enfermedad se manifiesta por sólo uno o dos de estos síntomas.

Habitualmente, el cólico biliar de la coledocolitiasis es complicado, de gran intensidad, y se asocia a estado nauseoso y vómitos rebeldes. Puede prolongarse durante varias horas y, si se alivia con antiespasmódicos, tiende a reaparecer precozmente. Se le puede confundir con un cólico ureteral derecho: es útil tener presente que en este caso, el paciente aparece muy inquieto, mientras que, si el dolor es de origen biliar, tiende a permanecer postrado en su cama. La ictericia puede ser fugaz o subclínica, manifestándose sólo por una coluria transitoria. En otros casos, el enclavamiento de un cálculo en la ampolla de Vater da origen a una ictericia prolongada, que en los enfermos ancianos puede complicarse con una enfermedad tubular aguda. La fiebre aparece en brotes aislados, precedidos por escalofríos; durante varios meses, estos episodios pueden constituir la única manifestación de la enfermedad. En los pacientes seniles, la coledocolitiasis suele ser causa de anorexia y de pérdida de peso, simulando una enfermedad neoplásica. Conviene insistir en que muchos enfermos con cálculos en el colédoco no tienen ningún síntoma que revele su presencia. Por ello es tan importante, durante la colecistectomía electiva o de urgencia, la cuidadosa exploración radiológica de la vía biliar. La incorporación rutinaria de este procedimiento ha reducido la incidencia de coledocolitiasis residual de un 10 por 100, a un 1 por 100 menos. Si durante una laparotomía se detecta la coledocolitiasis, se procede a la coledocostomía y limpieza del conducto, y se deja una sonda T de calibre adecuado para el drenaje postoperatorio. Cabe señalar que la coledocostomía aumenta significativamente la morbilidad y la mortalidad de la cirugía biliar. Rev Gastroenterol Mex. 2004 Nov;69 Suppl 3:112-6. Rojas O., Arizpe B., Marin L., Cesin S., San R. Rev Gastroenterol Mex. 2006 Jan-Mar;71(1):16-21. Gutierrez-Bermudez J.A., AdalidMartinez R., Guitron-Cantu A

17.- Diagnosticó un absceso hepático amibiano en un paciente masculino de 24 años. Indicó tratamiento con Metronidazol, con lo que se observó respuesta clínica favorable. Posteriormente complementa el tratamiento con el siguiente fármaco: a) b) c) d) l

Emetina Cloroquina Albendazol Iodoquinol (hidroxiquinoleína)

Posterior al tratamiento para la forma invasiva (trofozoito), para el cual el tratamiento más efectivo al momento es el Metronidazol, pero sobre el que tienen también efectos la

emetina, Cloroquina y tinidazol, se recomienda utilizar un agente con actividad cisticida (actividad luminal), como el iodoquinol o la Paromomicina. Kasper DL, Braunwald E, Fauci AS, Hauser SL, Longo DL, Jameson JL. Harrison´s Principles of Internal Medicine. McGraw Hill. 16 Ed. 1214-1217 pp.

18.- Un hombre de 47 años de edad afebril con antecedente de pancreatitis crónica, presenta una masa abdominal palpable y amilasa sérica persistentemente elevada. ¿La mayor probabilidad de diagnóstico es?

a) b) c) d)

Cistadenoma pancreático. Pseudoquiste pancreático. Carcinoma pancreático. Colección pancreática aguda

El pseudoquiste pancreático es una colección de jugo pancreático localizada, por lo general, en el interior o alrededor del parénquima pancreático. El pseudoquiste pancreático está confinado por una capa no epitelializada de tejido necrótico, fibrótico y de granulación, que se desarrolla tras una lesión pancreática. Para su formación requiere un mínimo de cuatro semanas desde que ese daño se produce. El pseudoquiste pancreático es una complicación tanto de la pancreatitis aguda como de la crónica. Si bien la mayoría de los pseudoquistes pancreáticos se localizan en la cabeza y el cuerpo del páncreas, hasta un 20% de los mismos son extrapancreáticos (1). Se han descrito pseudoquistes pancreáticos en múltiples localizaciones, como cavidad pleural, mediastino y pelvis (2). Se presenta el caso de un pseudoquiste pancreático de localización hepática que apareció en el curso de una agudización de una pancreatitis crónica, y que se resolvió sin necesidad de drenaje. Diagnóstico: Manifestaciones Clínicas: 1. 2.

Pacientes en la cuarta o quinta década de vida, antecedentes etiológicos. Pacientes con pancreatitis aguda que no resuelve luego de 5 a 7 días de tratamiento o luego de mejoría recae. 3. Sensación de cuerpo extraño y pesadez en la mitad superior del abdomen. 4. Si pancreatitis crónica, dolor abdominal o síntomas por compresión de víscera. 5. Nauseas, vómitos y pérdida de peso por obstrucción duodenal. 6. Ictero, si compresión del colédoco. 7. Masa en abdomen superior, lisa y dura, muchas veces insensible. 8. Más raramente ascitis y derrame pleural. Complementarios Diagnósticos: 1. 2.

Ultrasonografía Abdominal: Muchas veces diagnóstico, preferido para vigilancia. TAC: Ideal para diagnóstico.

Bibliografía 1. Hamm VB, Franzen N. Atypically located pancreatic pseudocyst in liver, spleen, stomach wall and mediastinum: their CT diagnosis. Rofo 1993; 159 (6): 522-7 2. Vitas GJ, Sarr MG. Selected management of pancreatic pseudocyst: Operative versus expectant management. Surgery 1992; 111 (2): 123-30. 3. Mofredj A, Cadranel JF, Dautreaux M, Kazerouni F, Hadj-Nacer K, Deplaix P, et al. Pancreatic pseudocyst located in the liver: a case report and literature review. J Clin Gastroenterol 2000; 30 (1): 81-3 4. Balzan S, Kianmanesh R, Farges O, Sauvanet A, O'toole D, Levy P, et al. Right intrahepatic pseudocyst following acute pancreatitis: an unusual location after acute pancreatitis. J Hepatobiliary Pancreat Surg 2005; 12 (2): 135-7.

19. - A 20-year-old man comes to the physician he has noticed blood in his urine on several occasions in the past year. Each episode of hematuria occurred in association with an upper respiratory tract infection or a flulike illness. Physical examination is unremarkable. A urine dipstick test shows mild proteinuria and microhematuria. Serum levels of electrolytes, creatinine, and blood urea nitrogen are within normal limits. Serum levels of IgA are elevated. Which of the following is the most likely diagnosis?

a) b) c) d)

Berger disease Goodpasture syndrome Henoch-Schönlein purpura Postinfectious glomerulonephritis

HEMATURIA GLOMERULAR AISLADA PERSISTENTE: Definición Presencia de hematuria de origen glomerular (con acantocitos, y ocasionalmente cilindros hemáticos), sin otro elemento de inflamación glomerular, es decir, sin HTA, ni edema, con proteinuria menor a 1 gm /24 hs, y función renal normal, estable. En caso de hematuria persistente, aislada, las posibilidades principales es que se trate de Glomerulonefritis por IgA (lo más frecuente), enfermedad de membrana basal fina, Sd. de Alport, (raro) o el inicio de una glomerulonefritis crónica (< probable) (15) .También puede tratarse de una GN Post-Streptocócica, con persistencia de hematuria. A continucación se describirán brevemente: Nefropatía por IgA: Enfermedad de Berger y síndrome de Shönlein-Henoch, se considerán por algunos como espectro de una misma enfermedad, siendo Berger la forma limitada al riñón.

Es la Glomerulonefritis aguda más frecuente. Se da entre los 15 y 35 años, y es 3 veces más frecuente en hombres, tiene tres patrones clínicos de presentación:

1. Como hematuria macroscópica, 24 – 48 hs. posterior a cuadro respiratorio alto, asociado a dolor lumbar. Hematuria dura 2 – 6 días, rara vez presenta HTA o deterioro de función renal, y tiene proteinuria mínima. Recurre hasta en un 50%. De esta forma se presenta un 40-50%. 2. Como hematuria microscópica, con proteinuria leve, detectada solo en Sed. Orina, por rutina. 30 – 40 % se presenta así. 3. Como Sd. Nefrítico (HTA, Edema, Hematuria), pudiendo evolucionar algunos de estos casos como GNRP. Se presentan así un 10 %. Generalmente cursan con proteinuria menor a 1-2 gm/24 hs,. Pocos desarrollan Sd. nefrótico (10 %)

Existe un grupo de pacientes (20 – 40 %), que evoluciona con falla renal progresiva, llegando a IRT en 5 – 25 años. Se ha detectado como factor de riesgo de esta evolución la presencia de Sd. nefrótico o proteinuria > 1gm/24h, , edad, HTA, deterioro de la función renal, ausencia de hematuria macroscópica. Si no tiene ninguno de estos factores, tiene poca probabilidad de desarrollar falla renal, y no se ha demostrado que el tratamiento, en este grupo de bajo riesgo, altere la evolución de la enfermedad. Al laboratorio, el complemento es generalmente normal, rara vez elevado. Existe aumento de IgA circulante entre un 30 – 50 %, sin ser específico de esta enfermedad. Para el diagnóstico, se requiere de biopsia renal, donde se identifican por inmunohistoquímica, la presencia de depósitos de IgA. Biopsia de piel carece de especificidad y de sensibilidad para diagnóstico de enfermedad de Berger. BIBLIOGRAFÍA 1.Jennette C, Falk R: Small Vesel Vasculitides. N Engl J Med 337:1512, 1997 2.Kashtan, CE Alport Síndrome and thin glomerular basement Membrane disease. JAM Soc Nephrol 1998;9:1736. 3.Falk R et al: Primary glomerular disease, en “Breneer & Rector´s, The Kidney, 6a ed, BM Brenner (editor). Phyladelfia, Saunders, 2000. pp 1263-1349. 4.Hricik, DE et al: Glomerulonephritis. N Eng J Med 339:889,1998 5.Antony BF. Attack rates of acute nephritis after type 49 streptoccocal infection of the skin and of the respiratory tract. J Clin Inv, 1969;48:1697. 6.Oliviera DBG. Poststreptococcal glomerulonephritis:getting to know an old enemy. Clin Rxp Immunol 1997;107:8-10 7.LangeK, et all. Evidence for the in situ origin of poststreptococcal glomerulonephritis: glomerular localization of endpstreptosin and the clinical significance of the subsequent antibody response. Clin Nephrol 1983;19:3-10 8.Rodriguez-Iturbe, B. Epidemic poststreptococcal glomerulonephritis, Kidney int 1984, 25:129. 9.Tejani A. Poststreptococcal glomerulonephritis: current clinical and pathologic concepts.

Nephron 1990;55:15. 10. Potter EV. Twelve to seventeen year follow up of patients with poststreptococcal acute glomerulonephritis in Trinidad. N Engl J Med 1982; 307:725-9.

20.- Un paciente de 36 años de edad acude a consulta por presentar nausea y vómito, aumento en el número de evacuaciones de consistencia aguada, (diarrea sin productos patológicos), afebril, T.a 110.70 mmhg, refiere que su hijo presentó los misma sintomatología casi al mismo tiempo que el, como antecedente refiere haber comido juntos en un restaurante hace aproximadamente 4 hrs ¿De los siguientes microorganismos cual es el de mayor probabilidad de ocasionar éste cuadro?

a) Salmonella enteriditis. b) Shigella sonnei. c) Staphylococcus aureus. d) E. Coli

La contaminación de alimentos por S. aureus, está asociada con una forma de gastroenteritis que se manifiesta clínicamente por un cuadro caracterizado por vómitos (76% de casos) y diarrea (77% de casos). El corto período de incubación de 1-6 horas orienta a la sospecha de enfermedad producida por ingestión de una o más enterotoxinas preformadas en el alimento que ha sido contaminado con cepas de S. aureus productor de la misma. Son raramente observados signos de toxicidad sistémica, tales como fiebre e hipotensión En general, es un cuadro autolimitado que típicamente se resuelve en 2448 horas desde el inicio. No está claro si se desarrolla en humanos inmunidad a largo plazo, pero anticuerpos frente a una SE no necesariamente confieren inmunidad frente a la intoxicación por S. aureus, ya que existe múltiples SE capaces de producir enfermedad. En algunos casos, anticuerpos producidos frente a una SE confieren protección cruzada contra otra SE, ya que algunas comparten epítopes. Todas las SE son capaces experimentalmente en primates de producir emesis, y no se registra enterotoxemia, excepto en dosis muy altas, probablemente debido a su dificultad para atravesar mucosas. El 99% de casos de intoxicación alimentaria por enterotoxinas Estafilocóccicas está asociado a S. aureus y ocasionalmente se reportan casos por Staphylococcus epidermidis. Las cepas estafilocóccicas enterotoxigénicas aisladas de alimentos implicados en brotes de infección son mas a menudo lisadas por fagos del grupo lll, y menos frecuentemente simultáneamente por los grupos l y lll.

Tratamiento. Como para la mayoría de enfermedades trasmitidas por alimentos autolimitadas, las medidas de sostén son la base del tratamiento. No está indicado tratamiento con antimicrobianos. Referencias bibliográficas.

- Manual of Clinical Microbiology. Murray, P. 1995 6th edition. - Principles and Practice of Infectious Diseases. Mandell, Douglas, Bennett. 1995. 4th edition. - Dinges M, Orwin P, Schlievert P. 2000. Exotoxins of Staphylococcus aureus. Clinical Microbiology Reviews, vol 13; 16-34

21.- Se trata de lactante de 6 meses que es llevado a control pediátrico de rutina. Los padres refieren que el niño llora frecuentemente, tiene un “carácter huraño”, se molesta mucho cuando lo exponen al sol y esconde su cara del mismo. La exploración muestra que hay fotosensibilidad extrema, córneas grandes de aspecto grisáceo y al tacto, el ojo está algo duro. Probable diagnóstico:

a) b) c) d)

Glaucoma congénito bilateral Glaucoma crónico de ángulo abierto bilateral Glaucoma agudo de ángulo cerrado bilateral Panuveitis bilateral

Los llamados glaucomas del desarrollo son un grupo de enfermedades caracterizadas por un defectuoso desarrollo del sistema de drenaje del humor acuoso y aunque el glaucoma puede no manifestarse hasta la edad adulta, la mayoría se presentan en la infancia. Desde que Collins (1) en 1893 y Cross en 1896 (2) determinaron como probable causa del glaucoma congénito la alteración en el desarrollo embrionario de las estructuras del ángulo iridocorneal, han sido muchos los que han llegado a la misma conclusión (3,4), en el sentido que sería una alteración en este desarrollo lo que determinaría una elevación en la presión intraocular y secundariamente el desarrollo del glaucoma. El desarrollo incompleto del segmento anterior puede llevar a distintas formas de glaucoma. Con el término de glaucoma congénito se ha designado clásicamente aquellos casos en los que el glaucoma se pone de manifiesto en los primeros años de vida. Si la alteración en el segmento anterior es más leve, provocaría una elevación de la presión más tardía, dando lugar al denominado glaucoma juvenil. Dentro de los glaucomas del desarrollo, el Glaucoma Congénito Primario es una enfermedad que se manifiesta en los primeros meses de la vida caracterizada por la existencia de una alteración en el desarrollo de la malla trabecular y estructuras angulares, no asociada a otras anomalías oculares o enfermedades sistémicas, y que va a condicionar una elevación patológica de la presión intraocular y secundariamente una lesión glaucomatosa del nervio óptico y alteraciones anatómicas en el globo ocular. TITULO: Glaucoma congénito bilateral. REFERENCIAS BIBLIOGRAFICAS 1. Collins ET. Congenital defects of the iris and glaucoma. TransOphthalmol. 1893; 13: 114. 2. Cross FR. Congenitalhidrophthalmos. TransOphthalmol Soc U K. 1896; 16: 340. 3. Lagrange F. Traitement du glaucoma infantile. Bull Soc OphtalmolFr. 1925; 38: 189. 4 Löhlein W. Das glaukom der jugendlichen. Albrecht v Graefes Arch Ophthal. 1913; 85: 393.

22.- Niño de 8 años de edad que presenta cuadro de ojo rojo de repetición en ojo izquierdo, y l visión disminuida a 20/80. El último cuadro tuvo 3 días de evolución. A la exploración se observa un ojo rojo moderado, apariencia despulida de la córnea y leve opacidad en área central. Tiene escasa secreción, lagrimeo y fotofobia acentuada. Al tocarla con un filamento de algodón, la córnea reacciona haciendo que se cierre el ojo más débilmente que le ojo contralateral. El diagnóstico más probable es:

a) b) c) d)

Herpes zoster Herpes simple Úlcera corneal Queratocono

Infección por herpes simple La manifestación inicial de la infección corneal por herpes simple (queratoconjuntivitis por herpes simple, queratitis) puede parecer una infección bacteriana leve porque los ojos están ligeramente doloridos, llorosos, rojos y sensibles a la luz. La inflamación de la córnea nubla la visión. Sin embargo, la infección por herpes no responde a los antibióticos, como haría una infección bacteriana, y suele empeorar cada vez más. Con mayor frecuencia, la infección produce sólo ligeros cambios en la córnea y desaparece sin tratamiento. En muy raras ocasiones, el virus penetra profundamente en la córnea, destruyendo su superficie. La infección puede ser recurrente, dañando aún más la superficie de la córnea. Las recurrencias pueden acabar en ulceración, cicatrización permanente y pérdida de la sensibilidad cuando se toca el ojo. El virus del herpes simple también puede provocar un incremento en el crecimiento de los vasos sanguíneos, empeoramiento de la visión o pérdida total de la misma. BIBLIOGRAFÍA 1. Maertzdorf J, Van der Lelij A, Baarsma GS, Osterhaus AD, Verjans GM. Herpes simplex virus type 1 (HSV-1)-induced retinitis following simplex encephalitis: indications for brain-to-eye transmission of HSV-1. Ann Neurol 2001; 49: 104-106. 2. Ahmadieh H, Sajjadi SH, Azarmina M, Kalani H. Association of herpetic encephalitis with acute retinal necrosis syndrome. Ann Ophthalmol 1991; 23: 215-219. 3. Kamel OR, Galloway GD, Trew DR. Delayed onset acute retinal necrosis 20 years following herpetic encephalitis. Eye 2000; 14: 788-789. 4. Pavesio CE, Conrad DK, McCluskey PJ, et al. Delayed acute retinal necrosis after herpetic encephalitis. Br J Ophthalmol 1997;81:415-416.

23.- Se trata de paciente 69 años, sin antecedentes neurológicos, psiquiátricos ni tratamientos farmacológicos previos, inicia aproximadamente hace 9 meses con deterioro

mental progresivo, fluctuaciones en su nivel de atención y rendimiento cognitivo, alucinaciones visuales y, en la exploración neurológica, signos parkinsonianos leves. El diagnóstico más probable es:

a) Enfermedad de Alzheimer. b) Demencia con cuerpos de Lewy. c) Enfermedad de Huntington. d) Demencia vascular

DIFERENCIAS ENTRE DCL Y EA Síntomas neuropsiquiátricos Alucinaciones visuales

+++ (precoces y persistentes)

+ (tardías)

Ideas delirantes

+++

++

Depresión

++

++

Apatía

++

++

Temblor

++

-

Rigidez

+++

+

Bradicinesia

+++

+

Signos motores extrapiramidales

Fluctuaciones clínicas Fluctuación cognitiva

+++ (Prominente, grave y precoz)

+

Neuropsicología Trastorno precoz en:

Atención, visuopercepción

Atención, memoria declarativa

Neuroimagen Atrofia cerebral global

++

++

Atrofia del lóbulo temporal medio

+

+++

Hipoperfusión del lóbulo occipital

+++

-

+++

-

++

+++

+

+++

Cuerpos de Lewy subcorticales

++

-

Cuerpos de Lewy corticales

+++

-

Déficit colinérgico

+++

++

Déficit dopaminérgico

++

-

++

++

Deterioro de la actividad dopaminérgica Neuropatología y bioquímica Densidad de placas seniles Densidad de ovillos neurofibrilares

Genética Sobrepresentación de APOE-ε4

+++ Manifestación típica ++ Habitualmente presente + Presente - Inusual Tabla adaptada de: U.P.Mosimann, I. McKeith. Dementia with Lewy bodies: diagnosis and treatment. Swiss Medical Weekly 2003;133:131-142 De la Vega, R. y Zambrano, A. Demencia con cuerpos de Lewy [en línea]. La Circunvalación del hipocampo, 2009

24.- Femenino de 28 años manifiesta que inicialmente presentó un enrojecimiento, sensibilidad y dolor en el borde externo del párpado. Actualmente cursa con orzuelo, usted decide el siguiente tratamiento por ser el de elección:

a) b) c) d)

Compresas frías Drenaje Compresas tibias y antibióticos tópicos Resección amplia

Orzuelo

Un orzuelo es una infección, en general provocada por un estafilococo, de una o más de las glándulas que se encuentran en el borde del párpado o por debajo de éste. Se forma un absceso que tiende a romperse y, en consecuencia, genera una pequeña cantidad de pus. El orzuelo a veces se forma al mismo tiempo que la blefaritis o bien como resultado de ésta. Una persona puede tener uno o dos orzuelos en toda su vida, pero otras los desarrollan repetidamente. El orzuelo en general se manifiesta primero con un enrojecimiento, sensibilidad y dolor en el borde externo del párpado. Luego, una pequeña área se torna redondeada y sensible y se hincha. El ojo puede lagrimear, volverse muy sensible a la luz intensa y provocar la sensación de que hay algo en su interior. Generalmente, sólo una parte muy pequeña del párpado se hincha, pero a veces se inflama en su totalidad. En general aparece un diminuto punto amarillento en el centro de la zona hinchada. A pesar de que se recurre a los antibióticos, no parecen ser demasiado útiles en estos casos. El mejor tratamiento consiste en aplicar compresas calientes durante 10 minutos varias veces al día. El calor ayuda a que el orzuelo madure, se rompa y drene. Cuando se forma un orzuelo en una de las glándulas más profundas del párpado, una afección llamada orzuelo interno, el dolor y los demás síntomas suelen ser más intensos. El dolor, el enrojecimiento y la hinchazón suelen aparecer sólo en un área muy pequeña, en general en el borde del párpado. Como esta clase de orzuelo rara vez se rompe por sí solo, el médico puede abrirlo para drenar el pus. Los orzuelos internos suelen ser recurrentes.

25.- Un niño de 6 años de edad refiere un desarrollo insidioso de “cojeo” con disminución en la movilidad de la cadera. Se queja ocasionalmente de dolor en la rodilla ipsilateral. Presenta una marcha anti-álgica. LA EF es normal, pero la movilidad pasiva de la cadera es limitada. El niño se encuentra afebril, y los padres refieren que su marcha y nivel de actividad eran normales hasta antes de este problema. No ha tenido fiebre recientemente. ¿Cuál es el diagnóstico más probable?

a) b) c) d)

Necrosis avascular de la epífisis capital femoral Displasia de cadera Osteomielitis hematógena de cabeza femoral Artritis séptica

La necrosis avascular de la cabeza femoral se manifiesta con dolor inguinal que se irradia de modo intermitente hacia la región anteromedial del muslo. Los Pacientes pueden presentar una marcha antiálgica, cojera por insuficiencia del glúteo menor o limitación del arco de movilidad, en especial para la flexión, la abducción y la rotación medial, con un chasquido provocado por la rotación lateral de la cadera en flexión y abducción, sobre todo al levantarse desde la posición sentada. Diagnóstico

El diagnóstico precoz requiere un índice de sospecha elevado en Pacientes que refieren dolor, en particular en caderas, rodillas u hombros. Las pruebas diagnósticas dependen de la fase en que esté el trastorno. La RM es la técnica más sensible y específica y se debe emplear para el diagnóstico en fases muy tempranas, cuando puede evitarse el colapso de la cabeza femoral o de otro hueso. Si no está clara la duración de la enfermedad, se deben obtener radiografías o TC para descartar enfermedad avanzada. La gammagrafía ósea es más sensible que la radiología, aunque inespecífica, por lo que se emplea menos que la RM. Los hallazgos radiográficos pueden no ser aparentes desde meses hasta 5 años después del comienzo de los síntomas. Los signos radiográficos comienzan con una esclerosis sutil del hueso. En la cabeza femoral se sigue de una radiotransparencia característica (el signo de la luna creciente); continúa con el colapso del hueso (aplanamiento de la cabeza femoral) y por último de estrechamiento del espacio articular y cambios artrósicos en los huesos enfrentados en la articulación. 1. López–Durán L. Necrosis óseas avasculares. En: López-Duran L. Traumatología y Ortopedia. 2ª ed. Madrid: Luzán, 1995; 5: 115-7. 2. Coombs RR, Thomas RW. Avascular necrosis of the hip. BrJ Hosp Med 1994; 51(6): 27580. 3. Mitchell MD, Kundel HL, Steimberg ME, Kressel HY,Alavi A, Axel L. Avascular necrosis of the hip: comparison of MR, Ct and scintigraphy. AJR Am J Roentgenol 1996;147 (1): 67-71. 4. Markisz JA, Knowles RJ, Altchek DW, Schneider R, WhalenJP, Cahill PT. Segmental patterns of avascular necrosi of the femoral heads: early detection with MR imaging.Radiology 1987; 162 (3): 717-20. 5. Mitchell DG, Kressel HY, Arger PH, Dalinka M, SpritzerCE, Steimberg ME. Avascular necrosis of the femoral head:morphologic assesment by MR imaging, with CT correlation.Radiology 1986; 161 (3): 739-42. 6. Manero FJ, Castellano MA, Manero I. Radiología ósea.Medicina Integral 1997; 30 (5): 217-9. 7. Dutkowsky JP. Trastornos no traumáticos diversos. En: Crenshaw AH. Campbell. Cirugía Ortopédica. 8ª ed. Buenos Aires: Panamericana, 1994; 1922-4. 8. Aaron RK, Lennox DW, Bunce GE, Ebert T. The conservative treatment of osteonecrosis of the femoral head: a comparison of core compresion and pulsing electromagnetic fields. Clin Orthop 1989; 249 (1): 209-18. 9. Rioja Toro J. Magnetoterapia. En: Rioja Toro J. Electroterapia y electrodiagnóstico. 2ª ed. Valladolid: Universidad de Valladolid 1996; 299-309. 10. Xenakis TA, Beris AE, Malizos KK, Koukoubis T, Gelalis J, Soucacos PN. Total hip arthroplasty for avascular necrosis and degenerative osteoarthritis of the hip. Clin Orthop 1997; 341: 62-8.

26.- El diagnóstico mas probable en un hombre de 60 años , con antecedentes de tabaquismo positivo de mas de 30 años de evolución , que presenta de manera brusca un síndrome miccional irritativo con tacto rectal normal, flujo urinario no obstructivo,

ecografía reno-vésico-prostática normal, sedimento urinario con microhematuria y urocultivo negativo, cistoscopia normal y citología urinaria con atipias, es:

a) Litiasis uretral. b) Esquistosomiasis. c) Carcinoma in situ vesical. d) Hipernefroma.

Ca vesical. La incidencia es mayor estadísticamente en los fumadores, especialmente de cigarrillos, debido a la acción de la nicotina en el metabolismo del triptofano por bloqueo en su degradación natural, acumulándose compuestos aromáticos con poder cancerígeno. Aumenta más en los grandes fumadores, aquellos que consumen más de 30 cigarrillos al día. Pero aún ante una exposición similar el riesgo varía entre las personas, lo que estaría relacionado con las vías metabólicas propias de cada individuo. Al dejar de fumar el riesgo disminuye lentamente los 20 años siguientes, pero nunca se llega al nivel basal de riesgo.

El cancer de vejiga es habitualmente de comienzo silencioso y la mayoría de las veces el diagnóstico se plantea por la presencia de hematuria macroscópica, el síntoma más característico. En otras oportunidades puede existir una hematuria microscópica reiterada y síntomas irritativos como disuria, urgencia miccional y polaquiuria. Clásicamente se ha establecido que se puede encontrar este cáncer en aproximadamente el 10% de los pacientes con hematuria. En el estudio del paciente con hematuria, especialmente en la edad de más riego, se debe solicitar un examen de orina con urocultivo, una ecotomografía renal y pelviana y/o una pielografía de eliminación. El examen de orina nos permite estudiar la existencia de una infección urinaria y también llevar a plantear otras alternativas diagnósticas. El estudio de imagen superior de ecotomografía y pielografía permite una evaluación completa de la vía urinaria, especialmente alta, donde por ser ésta una enfermedad que compromete todo el urotelio, se puede encontrar algún otro foco tumoral alto o existir dilataciones pieloureterales como consecuencia de obstrucciones que el crecimiento del tumor puede producir.

Posteriormente se debe efectuar una cistoscopía, el examen clásico y aún fundamental para el diagnóstico de esta patología. La cistoscopía permite la visualización completa de la vejiga, a través de la cual se evalúa el tumor en cuanto a su tamaño, número y ubicación y si es pediculado o sésil. Durante la cistoscopía es importante tomar muestras de biopsia adecuadas, tanto para clasificar el tumor como determinar el grado de infiltración de la pared vesical. En muchas ocasiones cuando se presentan tumores pequeños o medianos este procedimiento es diagnóstico y terapéutico a la vez, ya que la muestra que se extrae es la totalidad del tumor.

Para completar el estudio en los pacientes en que se sospecha un tumor invasor, se debe realizar una TAC de abdomen alto y pelviano. Este examen se realiza para determinar la extensión perivesical, para lo cual tiene una sensibilidad del 83% y especificidad del 82%. También nos puede informar de eventuales compromisos ganglionares o a más distancia. No tiene mayor utilidad en los tumores superficiales ni para determinar la invasión muscular de la pared vesical. Otra posibilidad que en forma excepcional puede utilizarse es la resonancia magnética, pero no tiene ventajas significativas sobre un estudio de TAC, excepto que podría ayudar a definir la extensión en capa muscular vesical.

Referencias 1. 2.

Benson R.C. Endoscopic managemente of bladder tumor. Urol.Cl.North Am.11,637, 1984

Boyd S.D., Skinner D.G. Quality of life of urinary diversion patients.J. of Urology 138,1836, 1989. 3. Bricker E.M. Bladder subsstitution after pelvic evisceration. Cli. North Am. 3o,1511,1950. 4. Catalona W.J., Ratliff T.L.BCG and superficial bladder cancer. Surgannu. 22,362, 1990 5. Catalona W.J. Urotelial tumors of the urinary tract. Campbell's Urology. Sounders Companym, Phyladelphia, 1994. 6. Hautmann, Hofstteter F., Franck F. Endoscopic neodyme Yag laser application in bladder tumors.Eur.Urol.7.278,1981. 7. Kock N.G. Ghoneim M.A. Urinary diversion via continent ileal reservoir.J. of Urol. 141,1111, 1989 8. Goodwin W.E. Harris >A.P. Open transcolonic ureterointestinal anastomosis. Surg.Ginecol.Ostetr. 97:295,1953.

27.- Un trabajador limpia vidrios con antecedente de fractura vertebral de C6 y fractura del extremo distal del radio derecho tras caída de un tercer piso se presenta a consulta, refiere dolor, limitación de la movilidad y hormigueo en los dedos 2º y 3º de la mano derecha, de predominio nocturno, que mejora durante el día. ¿Cuál de los siguientes diagnósticos es el más probable?: a) Síndrome del túnel del carpo. b) Radiculalgia C5-C6. c) Distrofia simpático refleja. d) Artrosis postraumática del carpo. El Síndrome del Túnel Carpiano (STC) es una patología que afecta a la mano, provocada por una presión sobre el nervio mediano a nivel de la muñeca. Esto provoca síntomas como adormecimiento y hormigueos en la mano (especialmente en los dedos pulgar, índice, corazón y mitad del anular). Puede existir dolor, que puede estar limitado a la mano y muñeca, pero que en algunas ocasiones se irradia hacia el antebrazo. El STC con frecuencia

despierta al paciente por la noche, y los síntomas pueden aparecer con actividades como conducir un vehículo, escribir, u otros ejercicios que suponen una utilización significativa de la mano. En el síndrome del túnel del carpo avanzado, puede producirse una pérdida de fuerza y una disminución de la masa muscular en la base del pulgar. CAUSA El túnel del carpo es un canal formado por los huesos de la muñeca y un ligamento (el ligamento transverso del carpo) situado en la cara palmar de la muñeca. Por este túnel transcurren todos los tendones que flexionan la muñeca y los dedos, y el nervio (nervio mediano) que recoge la sensibilidad del pulgar, índice, corazón y parte del anular y moviliza los músculos de la base del pulgar. Algunas personas nacen con túneles estrechos y por tanto están predispuestas a problemas de presión sobre el nervio. La utilización vigorosa de la mano, que conduce a una tendinitis de los tendones que flexionan el pulgar y los demás dedos, también puede conducir a un síndrome del túnel del carpo a través del engrosamiento de las vainas tendinosas. Las vainas engrosadas "rellenan" el túnel presionando sobre el nervio. Las personas con artritis reumatoide, hipotiroidismo, diabetes, amiloidosis, insuficiencia renal y algunos otros problemas médicos están más predispuestas a padecer este síndrome.

1.

2. 3. 4. 5.

Arthroshi I, Gummenson C, Johonsson R, Ornstein E, Ranstam J, Rossen I. Prevalence of carpal tunnel syndrome in a general population . JAMA. 1999;282:1538 Periódico El Público, 17 de octubre del 2007,pag 26 Durkan, JA. The carpal compression test: an instrumental device for diagnostic carpal tunnel syndrome. Lancet. 1990;335:393-5. Marshall S, Tardif G. Injection local of steroids in the carpal syndrome. Cochrane Data Base of Systematic Reviews. 2005. Issue 5. Verdugo RJ, Salinas RS, Castillo J, Cea JG. Tratamiento quirúrgico versus tratamiento no quirúrgico para el síndrome del túnel carpiano. Cochrane Data Base. 2005

28.- Al explorar a un paciente masculino de 40 años de edad quien presenta una otoscopía sin alteraciones, una hipoacusia derecha, con Rinne negativo derecho y positivo izquierdo y Weber con lateralización al lado derecho. ¿Usted piensa en?

a) Otitis serosa. b) Neuronitis vestibular). c) Hipoacusia súbita. d) Otosclerosis.

OTOSCLEROSIS

INTRODUCCION: OSTEODISTROFIA DE LA CAPSULA OTICA DEL LABERINTO.

CLINICA:  HIPOACUSIA:  HIPOACUSIA PROGRESIVA TRANSMISIVA UNILATERAL , EN LA CUAL LA PACIENTE ENTIENDE MEJOR EN AMBIENTES RUIDOSOS. (PARACUSIA DE WILLIS)  PROGRESIVA: AVANZA CON EL TIEMPO, EMBARAZOS.  TRANSMISIVA EN INICIO. LUEGO PUEDE HACERSE NEUROSENSORIAL.  UNILATERAL EN INICIO. LUEGO BILATERAL EN > DEL 75%  PARACUSIA DE WEBER: NO OYE CON MASTICACION.

  

VERTIGO : 10-20%. . SENSACION DE “BORRACHERA” OCASIONAL.

 

TAC: SI VERTIGO. SE VE FOCO OTOESPONGIOTICO.

ACUFENO: 75% . DE TONO GRAVE. MUY MOLESTOS. (ANSIOLITICOS)

ALTS PSIQUICAS: IRRITABILIDAD, INTROVERSION,.HIPOCONDRIACOS,...NEUROSIS. EXPLORACION:  OTOSCOPIA: NORMAL (MANCHA DE SCHWARTZE 10%)  ACUMETRIA: RINNE NEGATIVO. WEBER LATERALIZADO AL LADO MAS AFECTO SCHWABACH :ALARGADO.(VIA OSEA PACIENTE Y EXPLOR) (TRIADA DE BEZOLD) PR.GELLE: NEGATIVO (DIAPASON EN FRENTE—PRESION CON PERA DE POLITZER EN CAE.)  IMPEDANCIOMETRIA: CURVAS CENTRADAS Y EN OCASIONES ALGO APLANADAS. REFLEJO AUSENTE O INVERTIDO.(NO RECRUITMENT) AUDIOMETRIA: (+AUDIO VERBAL) ESTADIOS:  FORMA TIPICA O DE POLITZER-SIEBENMANN: LA MAS FRECUENTE. PROGRESION LENTA. 1) HIPOACUSIA DE TRANSMISION CON UMBRAL DE 20-40 Dbs,PEOR EN FRECUENCIAS GRAVES. 2) HIPOACUSIA MIXTA: TRANSMISION DE 40-60 DBS Y PERCEPCION DE 20-30DBS (ESCOTOMA EN 2000Hzs-MUESCA DE CARHART). 3) SORDERA CON GRAN COMPONENTE NEUROSENSORIAL DE 6080DBS OTOSCLEROSIS DE LERMOYEZ: HIPOACUSIA TRANSMISIVA PURA. OTOSCLEROSIS COCLEAR DE MANASSE: DESDE INICIO COMPONENTE NEUROSENSORIAL.PURA

Bibliografía  House J. Otosclerosis. Otolaryngol Clinics 1993;26(3):323-502.  Jerger J. Clinical experience with impedance audiometry. Arch Otolaryngol 1970;92:311

  

Lempert J. Improvement in hearing in cases of otosclerosis: A new, one stage surgical technique. Arch Otolaryngol 1938;28:42-97. Lippy WH, Schuring AG. Treatment of the inadvertently mobilized footplate. Otolaryngol Head Neck Surg 1973;98:80-81. Meyer S. The effect of stapes surgery on high frequency hearing in patients with otosclerosis Am J Otol 1999;20:36-40

29.- Femenino de 72 años con hipoacusia bilateral que ha ido progresando en los últimos años. Refiere oír pero no entender, especialmente cuando hay ruido ambiente. La audiometría tonal muestra una hipoacusia neurosensorial bilateral y casi simétrica para ambos oídos por afectación de los tonos agudos. Se comprueba una disminución de la inteligibilidad en el audiograma verbal. ¿Cuál es el diagnóstico más probable?:

a) Presbiacusia. b) Colesteatoma bilateral. c) Insuficiencia vertebrobasilar. d) Enfermedad de Ménière bilateral.

La presbiacusia (alteración de la audición relacionada con el envejecimiento) es la causa más frecuente de hipoacusia perceptiva en los adultos. Los factores que favorecen este trastorno son de tipo genético y ambiental, asociados sobre todo a la exposición al ruido. Existe una variabilidad interindividual en cuanto a la edad de aparición de la presbiacusia y a su evolución. El diagnóstico se establece ante la comprobación de una hipoacusia perceptiva bilateral y simétrica, de predominio en frecuencias agudas. La alteración de la inteligibilidad en la audiometría verbal es un elemento pronóstico. El tratamiento óptimo se basa en la adaptación audioprotésica bilateral y precoz. En algunos casos, la rehabilitación auditiva se favorece con una estrategia multidisciplinaria que incluya la rehabilitación logopédica y un posible apoyo psicológico. En ausencia de mejoría, la edad no representa una contraindicación para el implante de oído medio o coclear. Guerrero Medina, Enrique. Hipoacusia por Ruido. En: Seminario de Salud Ocupacional. . Memorias del III Seminario de Salud Ocupacional CAFAM, 1993. p. 6. Enrique, Escuela Colombiana de Medicina, 1993. Notas Técnicas de Prevención: Ruido: vigilancia epidemiológica de los trabajadores expuestos. Instituto Nacional de Seguridad e Higiene en el Trabajo . NTP - 193. 1988. Barcelona : INSHT. p. 2. ISSN-0212-0631. Notas técnicas de Prevención : Hipoacusia laboral por exposición a ruido: Evaluación clínica y diagnóstico. Instituto Nacional de Seguridad e Higiene en el Trabajo (España). NTP - 287. 1991. Barcelona: INSHT. p. 3.

30.- Ingresa al servicio de urgencias masculino de 47 años con hemorragia digestiva alta. No hay antecedentes de consumo de AINE. La endoscopia revela úlcera gástrica en incisura

angularis con un punto de hematina y mínimos restos de sangre oscura en el estómago. Se realizan biopsias del margen de la úlcera y una biopsia antral para prueba rápida de ureasa positiva. ¿Cuál de las siguientes actitudes es la más correcta?:

a) Iniciar con sucralfato y Ranitidina 150 mg/día,tratamiento anti-Helicobacter pylori si la histología confirma la presencia del germen. b) Se debe indicar anti-Helicobacter pylori durante 14 días, seguido de un antisecretor hasta que se confirme la erradicación del germen. c) Esclerosis endoscópica de la úlcera seguida de tratamiento con omeprazol, 20 mg/día durante 28 días. d) Iniciar tratamiento con Omeprazol 40mg/día durante 1 mes.

El Helicobacter pylori (HP) es una bacteria microaerófila, gramnegativa, de crecimiento lento y forma helicoidal con abundantes flagelos. Fue descubierta por dos médicos autralianos. Robin Warren y Barry Marshall; trabajando en colaboración, detectaron que este microorganismo se encontraba en casi todos los pacientes con inflamación gástrica, úlcera duodenal o gástrica. Basándose en estos resultados propusieron que HP estaba implicado en la etiología de estas enfermedades. Antes de 1982, se pensaba que la mayor causa de la úlcera péptica era el estrés y el estilo de vida. Ahora se sabe que HP está implicado en más del 90% de las úlceras duodenales y hasta el 80% de las úlceras gástricas. Gracias a los descubrimientos de Marshall y Warren, la úlcera péptica no es una enfermedad crónica sino que puede ser curada con una pauta de tratamiento con antibióticos y con inhibidores de la secreción ácida. Afecta al 50 % de la población mundial, ha sido identificado como el agente causal de la úlcera péptica y se ha clasificado además como carcinógeno tipo I. Como resultado de su interferencia con la secreción de ácido por el estómago, esta bacteria es capaz de generar deficiencias en la absorción de nutrientes y vincularse con la aparición de manifestaciones carenciales o con el agente causal de enfermedades crónicas

El objetivo del tratamiento médico será promover la cicatrización de la úlcera para prevenir la recurrencia de la hemorragia. Así, la ranitidina puede prevenir la úlcera duodenal en pacientes tomadores de AINES, pero es ineficaz en la prevención de la ulcera gástrica. Por el contrario, la famotidina protege la mucosa gástrica contra tratamientos cortos de aspirina o naproxeno. Si se demuestra que existe infección por H.Pylori estará indicada su erradicación para evitar recurrencias. Amoxicilina + Klaritromicina OD+ esomeprazol o lanzoprazol Erradicación en 14 dias 100% De existir una complicación, como la hemorragia digestiva, los pacientes deberán ser internados, suprimir el aporte oral, efectuar el diagnóstico de hemorragia, determinar la cuantía de la pérdida hemática y realizar un adecuado control de la hemodinamia

Lahaie RG, Gaudreau C. Helicobacter pylori antibiotic resistance: trends over time. Canadian Journal of Gastroenterology. 2000;14(10):895–899.

Manes G, Balzano A, Iaquinto G, et al. Accuracy of the stool antigen test in the diagnosis of Helicobacter pylori infection before treatment and in patients on omeprazole therapy. Alimentary Pharmacology and Therapeutics. 2001;15(1):73–79. McManus TJ. Helicobacter pylori: an emerging infectious disease. Nurse Practitioner. 2000;25(8):42–46.

31.- Masculino de 42 años con antecedentes de hipertensión arterial sistémica tratado con losartan 50 mg v.o. actualmente controlado, inició con sintomatología urinaria asociado a dolor en fosa renal izquierda, se diagnostica un cálculo de 1cm ubicado en la pelvis renal, el tratamiento ideal para este paciente es : a) b) c) d)

Pielolitomía Plastía ureteropiélica Litotricia electro hidráulica Litotripsia extracorpórea

La litotripsia extracorpórea constituye el tratamiento en la actualidad más utilizado para la litiasis urinaria. Las contraindicaciones absolutas para este procedimiento son: embarazo, obstrucción distal y la presencia de infección activa. Presenta como limitación cálculos no localizables debido a su pequeño tamaño (menor 2-5mm) Manual CTO de Medicina y Cirugía. Séptima Edición. McGraw Hill, pgs 1637-1642.

32.- Femenino diabética e hipertensa de 70 años con antecedente de constipación crónica y dolor abdominal intermitente de tipo cólico, refiere haber tenido evacuaciones con sangre roja en escasa cantidad y en forma esporádica desde hace 6 meses. El diagnostico más probable es:

a) b) c) d)

colon irritable. cáncer del recto. colitis amibiana. enfermedad diverticular.

Divertículo: • Protrusión sacular de la mucosa a través de la pared muscular del colon [25] • La protrusión ocurre en las áreas débiles de la pared intestinal donde pueden penetrar los vasos sanguíneos • Habitualmente 5–10 mm de tamaño

• Los divertículos son realmente pseudodivertículos (falsos divertículos), ya que contienen sólo mucosa y submucosa cubiertas de serosa Enfermedad diverticular Consiste en: • Diverticulosis – presencia de divertículos dentro del colon • Diverticulitis – inflamación de un divertículo • Sangrado diverticular Generalmente, el cuadro de diverticulitis aguda se presenta en pacientes de más de 40 años, se inicia con dolor en el cuadrante inferior izquierdo (93-100%) que se puede generalizar a todo el abdomen según la evolución. El dolor suele acompañarse de peritonismo y se puede acompañar de náuseas y vómitos, así como alteraciones del tránsito intestinal (estreñimiento/diarreas). La sintomatología urinaria que presenta en ocasiones se debe habitualmente a la irritación que produce la masa inflamatoria en la vecindad del aparto genitourinario. La exploración abdominal presenta amplias variaciones, desde un cuadro anodino a una peritonitis difusa con alteraciones sistémicas, pasando por el dolor en fosa ilíaca izquierda. La fiebre (57-100%) y la leucocitosis (69-83%), sin ser constantes, son útiles en el seguimiento del paciente24. Mención aparte merecen los casos de diverticulitis complicada

Diverticulitis y sangrado Si la enfermedad diverticular causa un sangrado activo y severo, se debe resecar el segmento involucrado76-79. Para localizar el punto del sangrado se pueden utilizar el lavado colónico intraoperatorio y la endoscopia80. En ocasiones no se consigue localizar el segmento causante del sangrado, en cuyo caso está indicada la colectomía subtotal81. Se puede intentar el tratamiento endoscópico, que ha demostrado ser efectivo82,83, considerando la posibilidad de efectuar una cirugía electiva posterior.

Enfermedad diverticular: revisión histórica y estado actual M. Rodríguez*,V. Artigas*, M.Trías*, J.V. Roig** y R. Belda*** Servicios de Cirugía General y Digestiva. *Hospital de la Santa Creu i Sant Pau. Barcelona. **Hospital de Sagunto. ***Hospital de Torrecárdenas.

33.- Varón de 28 años de edad con AV en los dos ojos de 20/30. Al explorarlo el paciente entrecierra los ojos. De cerca ve bien, pero se cansa al estar leyendo después de un tiempo. El resto de la exploración es normal. ¿Cuál es el diagnóstico más probable? a) Miopía

b) Astigmatismo c) Hipermetropía d) Miopía y presbiopía

AMETROPÍAS Cuando los rayos de luz paralelos que inciden en un globo ocular enfocan a nivel de la retina, dicha condición se conoce como emetropía. Si, por el contrario, estos rayos de luz no enfocan a nivel de la retina, se habla entonces de ametropía. En esta última el foco principal se encuentra por delante (MIOPÍA) o por detras de la retina (HIPERMETROPÍA), tanto más retirado de ella cuanto mayor sea la ametropía, o bien pierde su característica puntiforme para formar dos líneas focales principales perpendiculares entre sí (ASTIGMATISMO). El astigmatismo es un defecto refractivo que hace que todo se vea deformado o desenfocado, tanto en visión cercana como en visión lejana. En el ojo astígmata, la córnea tiene forma de elipse, esto hace que las imágenes no enfoquen en un foco único como en el ojo normal. El astigmatismo puede combinarse con miopía, dando lugar a un astigmatismo miópico donde además de ver desenfocado de lejos, las imágenes tanto cercanas como lejanas se perciben deformes. El astigmatismo puede combinarse con hipermetropía, dando lugar a un astigmatismo hipermetrópico donde además de ver desenfocado de cerca, las imágenes tanto cercanas como lejanas se perciben deformes. El síntoma más importante del astigmatismo es la percepción de imágenes distorsionadas. Así mismo son habituales los dolores de cabeza frontales y en algunas ocasiones inclinaciones laterales de la cabeza compensatorias. El astigmatismo distorsiona o borra la visión a una cierta distancia, tanto de lejos como de cerca. La visión es similar al efecto de los espejos deformados, los cuales reproducen objetos demasiado altos, demasiado anchos o demasiado delgados

PAC Oftalmologia Parte B Libro 4 Errores Refractivos y Cirugía Refractiva GENERALIDADES ÓPTICAS

34.- Masculino de 66 años con antecedentes de constipación y dolor abdominal en hipogastrio y fosa ilesa izquierda de manera recurrente. El enema baritado muestra múltiples divertículos en el sigmoides. ¿Cuál es el tratamiento más apropiado?

a) b) c) d)

Colonoscopía con biopsia Ranitidina Enema con esteroides Dieta alta en residuos

Tratamiento El tratamiento de la diverticulosis intenta reducir el espasmo segmentario. Una dieta rica en residuos es de utilidad y puede suplementarse con preparados de semillas. Las dietas bajas en residuos están contraindicadas. Teóricamente deberían ser de ayuda los antiespasmódicos (p. ej., la belladona); su valor en la práctica es difícil de enjuiciar. Su utilización crónica, especialmente en los ancianos, suele causar efectos secundarios adversos. No está justificada la cirugía en caso de enfermedad diverticular sin complicaciones. La resección con anastomosis del área del intestino afectada en la colitis espástica (una combinación de divertículos, espasmo y diarrea) puede llevar a resultados equívocos. Deben cateterizarse las arterias mesentéricas superior e inferior. La inyección selectiva de vasopresina controla la hemorragia en el 70% de los Pacientes. En algunos casos la hemorragia recurre a los pocos días y es necesaria la cirugía. Es posible practicar la resección segmentaria si se conoce el punto sangrante; en alrededor del 75% de los Pacientes este punto se encuentra en una zona proximal respecto a la flexura esplénica, aun cuando los divertículos predominen en el lado izquierdo. Si no se puede identificar el punto sangrante está indicada una colectomía subtotal. Los divertículos gigantes deben ser quirúrgicos. Estas lesiones pueden observarse en las radiografías abdominales simples o demostrarse mediante enema de bario. Dado que la posibilidad de infección o perforación es alta, se prefiere la resección del área afectada del colon. Blibliografía Shakelford’s. Surgery of the alimentary tract. 5a. Ed. 2002. Tomo 3. Feldman´s. Gastroeneterology. 2002. Perez. Anatomía y fisiología del hígado. Univ. Católica de Chile. 2005. Bratiz. Serum laboratory test in cirrhosis. Journal of Hepatology. Slovakia. 2005. Paradis. Glycomics. Journal of hepatology. Ireland. Agosto 2005.

35.- Masculino de 52 años, el cual presenta distensión abdominal en incremento e ictericia de 6 semanas de evolución. La EF revela arañas vasculares, venas ingurgitadas alrededor del ombligo y ascitis. ¿Cuál es la causa más probable de esta ictericia?

a) b) c) d)

Hepatitis Coledocolitiasis Cáncer de páncreas Cirrosis

1.- CIRROSIS HEPÁTICA DEFINICIÓN: Se trata de una alteración crónica e irreversible que representa la etapa final de diversos procesos agudos o crónicos que afectan al hígado, si bien el concepto es fundamentalmente morfológico donde se define como una alteración difusa de la arquitectura del hígado por

fibrosis y nódulos de regeneración, clínicamente se puede sospechar por los antecedentes y los diversos datos clínicos que se pueden documentar. Se sabe que los cambios histológicos condicionan una alteración vascular intrahepática y una reducción de la masa funcional hepática y como consecuencia se desarrolla hipertensión portal e insuficiencia hepática, dando como resultado final sus complicaciones mayores: ascitis, hemorragia digestiva, ictericia y encefalopatía hepática.

• •

• •

Cirrosis: Proceso hepático difuso con fibrosis y formación de nódulos Muchas causas; resultado final similar – Necrosis – Fibrosis I III – Regeneración Disminución síntesis Alteraciones vasculares: Hipertensión portal

ALCOHOL • Es una de las principales causas de cirrosis • El consumo per capita está correlacionado a la mortalidad global por cirrosis • Dependencia de alcohol en 5% de la población • Umbral de riesgo: 60 g/d en hombres, 20 g/d en mujeres ETIOLOGÍA: Consumo excesivo de alcohol. Infección crónica por los virus de la hepatitis B y C. Hemocromatosis hereditaria Enfermedad de Wilson Hepatitis autoinmune Esteatohepatitis no alcohólica. Colestasis obstructivas crónicas Obstrucción crónica del drenaje venoso Toxicidad por fármacos (Metotrexato, metildopa, vitamina A, etc.). En los niños la causa más frecuente es la anormalidad congénita de la vía biliar (atresia de vía biliar, enfermedad de Alagille, enfermedad de Byler). Cuando no es posible determinar el factor etiológico (5%) se denomina cirrosis criptogenética. CLÍNICA: La cirrosis compensada puede ser clínicamente latente, asintomática y ser descubierta fortuitamente en un examen clínico efectuado por otro motivo. Otros pacientes se diagnostican cuando son explorados por síntomas inespecíficos, o en el seguimiento de una hepatitis crónica viral, en algunos otros casos los datos son muy floridos aunados a los antecedentes de relevancia (ingesta intensa de alcohol). Los signos y síntomas que pueden acompañar a la cirrosis son múltiples y muy variados, si bien no todos se presentan en todos los casos ni tiene un carácter patognomónico se pueden observar en la gran mayoría de estos enfermos; los podemos clasificar por aparatos y sistemas: SIGNOS Y SÍNTOMAS DE LA CIRROSIS HEPÁTICA Síntomas generales:

Astenia Adinamia Signos cutáneos-ungueales Arañas vasculares Telangiectasias Eritema palmar Hipertrofia parotídea Contractura de Dupuytren Acropaquia Uñas en vidrio de reloj Desaparición de la lúnula (leuconiquia) Exploración abdominal Hepatomegalia Esplenomegalia

36.-Un varón de 30 años presenta dolor y tumefacción del testículo derecho. Su médico solicita un ultrasonido que revela una masa testicular de 2 x 2.5 cm. Se realizan una exploración inguinal y una orquiectomía. El estudio histopatológico reveló un seminoma puro. Una tomografía computadorizada de tórax, abdomen y pelvis mostró dos ganglios retroperitoneales de 3 cm que están aumentados de tamaño. La biometría hemática, la química sanguínea y los marcadores tumorales están todos dentro de los límites normales. ¿Cuál de los siguientes seria la mejor conducta? a) b) c) d)

Extirpación quirúrgica de toda la enfermedad Radioterapia. Braquiterapia Observación

Allen R. M. MMS Medicina Interna. 5ª. Edición. National Medical Series. Mc. Graw Hill. 2006. (capítulo 4IX G 2). El cáncer testicular es el más común en varones adultos jóvenes variedades más frecuentes son seminomas y tumores de células germinales no seminomatosos; ambos sor rabies aun en etapas avanzadas. Los seminomas son muy sensibles a la radioterapia; por tanto, los pacientes enfermedad de etapa II (la limitada a testículo y a ganglios por abajo del diafragma) pueden tratarse con < bajas de radiación. Debido a la toxicidad de la médula ósea producida por la radioterapia mediastínica, y está indicada la radiación profiláctica del mediastino. En este contexto, los ganglios linfáticos retroperito les aumentados de tamaño denotan la existencia de enfermedad metastásica y está indicado el tratamienl 37.- Masculino de 77 años portador de virus C y diabético desde hace 10 años, que presenta lesiones ampollosas de contenido serohemorrágico en extremidades superiores e inferiores, y erosiones a nivel de la mucosa bucal. La biopsia cutánea muestra una ampolla subepidérmica, se realiza inmunofluorescencia directa que muestra la presencia de depósitos de IgG a nivel de la membrana basal. El diagnóstico más probable es: a) Dermatitis herpetiforme. b) Penfigoide ampolloso.

c) Toxicodermia ampollosa. d) Porfiria cutánea tarda.

Penfigoide ampolloso Casi siempre afecta a mayores de 60 años. Se trata de una enfermedad ampollosa crónica autoinmune causada por la unión de autoanticuerpos tipo IgG y complemento C3 al antígeno del penfigoide, una glucoproteina transmembrana de 180 (antígeno menor) o 230 kD (antígeno mayor, desmoplaquina I) situada en los hemidesmosomas del queratinocito basal, a nivel de la lámina lúcida. Aunque se había dicho que el penfigoide podía ser paraneoplásico, existen estudios epidemiológicos que demuestran que el incremento de carcinomas asociados se debe unicamente a que ambas patologías, el penfigoide y el cáncer, se dan con mayor frecuencia al mismo grupo de edad. Patogénesis Se cree que la unión de los auto anticuerpos IgG al antígeno del penfigoide produce activación del complemento, lo que da lugar a la síntesis de C3a y C5a que causan desgranulación de los mastocitos. Éstos a su vez liberan mediadores entre los que destaca el factor quimiotáctico de los eosinófilos. Los eosinófilos reclutados liberan enzimas lisosomales que destruyen los hemidesmosomas y los filamentos de anclaje con lo que se separa la unión dermoepidérmica en la lámina lúcida. Clínica Se caracteriza por lesiones ampollosas grandes y tensas de contenido seroso o serohemático, localizadas o generalizadas, con distribución preferente en flexuras y que curan sin dejar cicatriz residual. La aparición de ampollas puede ir precedida de un cuadro urticarial o eczematoso. Pueden ser o no pruriginosas. En cerca del 30% de los casos existe afectación mucosa (generalmente mucosa oral).

Penfigoide ampolloso Diagnóstico El diagnóstico se establece por criterios clínicos, histológicos e inmunopatológicos. En el estudio histológico de la biopsia cutánea de una ampolla se observa una hendidura subepidérmica en el interior de la cual se aprecia un rico infiltrado inflamatorio mixto con predominio de eosinófilos que también abundan en la dermis.

Mediante inmunofluorescencia directa se comprueba la existencia de depósitos lineales de IgG y/o C3 en la región de la membrana basal. La inmunofluorescencia indirecta, utilizando fluido de la ampolla u orina, pone de relieve en cerca de la mitad de los pacientes la existencia de IgG y/o C3 circulantes. Bibliografía 1. Worjnarowska F, Kirtschig G, Highet AS, Venning VA, Khumalo NP. Guidelines for the management of bullous pemphigoid. Br J Dermatol 2002; 147:214-221. 2. Downham TF, Chapel TA. Bullous pemphigoid. Arch Dermatol 1978; 114:1639-1642. 3. Chosidow O, Saas V, Diquet B. Correlation between the pretreatment number of blisters and the time to control bullous pemphigoid with Prednisone 1mg/Kg/day. Br J Dermatol 1992; 127:185-195. 4. Fiveson D, Breneman D, Rosen G et al. Nicotinamide and tetracycline therapy of bullous pemphigoid. Arch Dermatol 1994; 130:753-758. 5. Fleming TE, Korman NJ. Cicatricial pemphigoid. J Am acad Dermatol 2000; 43:571-591. 6. Eisen D, Ellis CN, Voorhees JJ. Topical Cyclosporine for oral bullous disorders. J Am Acad Dermatol 1990; 23:936-937.

38.- .- Una mujer de 32 años de edad es llevada a quirófano por una laparoscopía diagnóstica debido a dolor pélvico y en cuadrante superior izquierdo crónicos (durante los últimos 2 años). No tiene alteraciones funcionales vesicales o intestinales. Tiene antecedente de 2 episodios de gonorrea previos. Bebe una cerveza al día. Labs: HCG urinaria negativa; Hto 39%; Leuc. T 8 000; Plt 200 000; AST 12; ALT 14. Intraoperatoriamente se observan adhesiones densas que involucran los oviductos, ovarios y útero. También se observan adhesiones perihepáticas que se extienden desde la superficie hepática hacia el diafragma. ¿Cual de los siguientes es el diagnóstico más probable? a) b) c) d)

Sx de Fitz-Hugh-Curtis Hepatitis Carcinoma hepatocelular Sx Wolff-Parkinson-White

El síndrome de Fitz-Hugh-Curtis se define como la presencia de una perihepatitis asociada a salpingitis. Los agentes etiológicos reconocidos hasta la fecha son Chlamydia trachomatis y Neisseria gonorrhoeae. El cuadro clínico de este síndrome es inespecífico y puede ser confundido con procesos inflamatorios o infecciosos del tubo digestivo, aparato urinario y respiratorio, en los cuales la manifestación sintomática fundamental es el dolor en hipocondrio derecho. El diagnóstico debe de sospecharse en aquella mujer joven con vida sexual activa que tenga antecedentes de promiscuidad en ella o en su pareja, que se queje

de dolor subcostal derecho. Es más probable el diagnóstico si se cuenta con el antecedente de enfermedad pélvica inflamatoria y más aún, si se tiene evidencia de que ésta sea causada por Neisseria gonorrehoeae y/o Chlamydia trachomatis. El diagnóstico definitivo se realiza con la visualización directa de la adherencia perihepática por laparoscopía o laparotomía. Se recomienda la primera. El tratamiento médico es a base de cefalosporinas y dicloxacilina y en algunos casos se requiere de la extirpación quirúrgica del proceso adherencial para mitigar el dolor. El Síndrome de Fitz-Hugh-Curtis. Causa frecuente de error de diagnóstico en hepatología y gastroenterología / The Fitz-Hugh-Curtis Syndrome. a frequent misdiagnosis in hepatology and gastroenterology Rev. gastroenterol. Méx;60(4):223-8, oct.-dic. 1995.

39.- Se trata de paciente femenino de 6 años de edad la cual presenta ceguera nocturna (hemeralopia) en la exploración oftalmológica se observa constricción del campo visual con escotoma anular, pérdida de la agudeza y electrorretinograma anómalo asi mismo arterias estrechadas ¿Cuál de las siguientes patologías es la más probable?:

a) Retinoblastoma. b) Catarata congénita o infantil. c) Retinosis pigmentaria (retinitis pigmentaria). d) Persistencia de vítreo primario

Retinosis pigmentaria. Bilateral, de curso lento y progresivo, comienza en la edad escolar, pudiendo causar ceguera hacia los 40 años. Es una alteración de los bastones. Puede presentarse aislada o asociada a otras malformaciones, como la polidactilia, el síndrome de Laurence-Moon-Bield. Se hereda bajo diferentes patrones. Clínicamente presentan mala visión nocturna (hemeralopía) y escotoma anular en el campo visual .Oftalmoscópicamente se aprecian arterias estrechadas, atrofia de papila y acumulaciones de pigmento que asemejan osteocitos en la retina periférica. Evolucionan hacia una reducción del campo visual y finalmente ceguera. Además presentan complicaciones adicionales como aparición temprana de cataratas y glaucoma.

Retinosis pigmentaria

Retinosis pigmentaria, preguntas y respuestas, Universidad Miguel Hernández de Elche, 2007 Science Daily (ed.): «Retina Transplants Show Promise In Patients With Retinal Degeneration» (11-7-2008).

40.- Una mujer de 17 años tiene exantema cutáneo rojo difuso; fiebre de 39.4°C y diarrea leve acuosa. En fechas recientes tuvo infección de garganta por la que se le administró sulfametoxazol. Comenzó sus menstruaciones hace tres días. En la exploración física se encuentran cambios eritematosos difusos de la piel con descamación temprana. La boca y las conjuntivas están eritematosas. ¿Cuál de los siguientes explica todo el proceso? a) b) c) d)

Síndrome de choque tóxico (TSS) Tuberculosis Mononucleosis por virus de Epstein-Barr Alergia a sulfametoxazol

Allen R. M. MMS Medicina Interna. 5ª. Edición. National Medical Series. Mc. Graw Hill. 2006. (Capítulo 8 VII D 1 a, 2 b). La bacteriemia por Salmonella, el síndrome de choque tóxico (TSS), la tuberculosis y la mononucleosis de Epstein-Barr pueden acompañarse de fiebre, pero la presencia de exantema descamativo difuso sugiere TSS, reacción farmacológica grave (p. ej., síndrome de Stevens-John-son); enfermedad de Kawasaki, o escarlatina. El exantema cutáneo relacionado con salmonelosis es muy sutil y evanescente (manchas de color rosa). La tuberculosis no se caracteriza por afección cutánea difusa y de mucosas o diarrea acuosa. Si bien la alergia al sulfametoxazol puede producir eritema cutáneo y de mucosas, no causa diarrea.

41.- Se trata de femenino de 75 años que desde hace dos semanas presenta ictericia. Hace dos días se añade confusión mental. A la EF FR 20, FC 110, TA 90/60, Temperatura 39°, no responde a comandos verbales, pero se aleja del dolor que se provoca al palpar el hipocondrio derecho y epigastrio. Los datos clínicos en el caso anterior son sugestivos de:

a) b) c) d)

Coledocolitiasis Cáncer de páncreas Colangitis Cirrosis

Diagnóstico El diagnóstico de colangitis se basa en la asociación de signos y síntomas de infección con los propios de una obstrucción biliar. La presentación clásica es la aparición de dolor en hipocondrio derecho o epigastrio junto a fiebre, generalmente alta, con escalofríos, e ictericia (tríada de Charcot). Cuando se añade confusión mental y shock (sepsis) se denomina Pentada de Reynolds, que se observa con menor frecuencia, pero habitualmente se asocia con una colangitis supurada grave. Sin embargo, la correlación entre la clínica

típica, las formas atípicas y la presencia de pus en la vía biliar es pobre y en muchos casos de colangitis faltan algunos de estos rasgos. Algunos enfermos, sobre todo de edad avanzada, pueden tener confusión mental o shock sin fiebre, o existir una leucocitiosis con desviación izquierda como única manifestación de la infección, por lo que se debe sospechar la existencia de una colangitis subyacente, sobre todo en enfermos de edad con algunas de estas manifestaciones. El laboratorio muestra hallazgos de obstrucción biliar con aumento variable de la bilirrubina y enzimas de colestasis. Es habitual encontrar leucocitosis con desviación izquierda, siendo por lo general las cifras más altas, en torno a 20.000 por mm 3 , reflejo de las formas más graves. La ecografía es la técnica de elección para detectar la existencia de obstrucción biliar por su elevada eficacia y versatilidad, pudiéndose detectar también complicaciones de la colangitis, como el absceso hepático. Dentro de la colangitis se pueden establecer unas formas leves, generalmente de buen pronóstico, y unas formas graves, donde se concentra la mayor parte de la mortalidad. Las primeras se manifiestan como cuadros febriles sin signos de afectación sistémica que se autolimitan espontáneamente o bajo tratamiento médico en 24-48 horas. Las formas inicialmente graves son las que asocian confusión mental, hipotensión, shock o fracaso renal. La edad es un factor constante de aumento de la morbimortalidad. Referencias bibliográficas 1. Bilhartz LE, Horton JD. Gallstone disease and its complications. En: Gastrointestinal and liver diseases. Sleisenger and Fordtran. Filadelfia: WB Saunders Co., 1998; 948-972. 2. Chung-Mau L, Chi-Leung L, Lai ECS, Sheuns-Tat F, Wong J. Early versus delayed laparoscopic cholecystectomy for treatment of acute cholecystitis. Ann Surg 1996; 223: 37-42. 3. Hamy A, Visset J, Likholatnikov D, Lerta F, Gibaud H, Savigny B et al. Percutaneus cholecystostomy for acute cholecystitis in critically ill patients. Surgery 1997; 121: 398401. 4. Harris A, Chong Hen Chang A, Torres-Viera C, Hammett R, Carr-Locke D. Meta-analysis of antibiotic prophylaxis in endoscopic retrograde cholangiopancreatography (ERCP). Endoscopy 1999; 31: 718-724. 5. Hermann RE. Surgery for acute and chronic cholecystitis. Surg Clin North Am 1990; 70: 1.263-1.275. 6. Koo Kp, Thirlby RC. Laparoscopic cholecystectomy in acute cholecystitis. What is the optimal timing for operation? Arch Surg 1996; 131: 540-545. 7. Lai ECS, Mok FPT, Tan ESY, Lo CM, Fan ST, You KT et al. Endoscopic biliary drainage for severe acute cholangitis. N Engl J Med 1992; 326: 1.582-1.586. 8. Marton KI, Doubilet P. How to image de gallbladder in suspected cholecystitis. Ann Int Med 1988; 109: 722-727. 9. Van den Hazel SJ, Speelman P, Tytgat GNJ, Dankert J, Van Leeuwen DJ. Role of antibiotics in the treatment and prevention of acute and recurrent cholangitis. Clin Infect Dis 1994; 19: 279-286.

10. Westphal J-F, Brogard J-M. Biliary tract infections. A guide to drug treatment. Drugs 1999; 57: 81-91.

42. - A 35-year-old man complains to a physician of chronic vague gastric pain of several years’ duration. The pain is sometimes relieved by food. Serum immunoglobulin studies for IgG and IgA antibodies directed against Helicobacter pylori are strongly positive. Endoscopy with gastric antral biopsy demonstrates gastrids but no ulcerative lesions. H. pylori organisms are seen with special stains on the biopsy fragments. The patient is treated with a 1-week course of omeprazole (20 mg bid), plus clarithromycin and metronidazole (500 mg bid each). Which of the following is the most appropriate test to noninvasively determine whether the H. pylori has been eradicated?

a) b) c) d)

Culture of gastric biopsy Repeat qualitative IgA and IgG anjtibodies against H. pylori Repeat quantitative IgA and IgG antibodies against H. pylori Urea breath test

Lahaie RG, Gaudreau C. Helicobacter pylori antibiotic resistance: trends over time. Canadian Journal of Gastroenterology. 2000;14(10):895–899.

43.- Se trata de paciente masculino de 70 años de edad sin antecedentes de importancia, que acude a consulta por presentar desde hace 6 meses malestar abdominal alto durante la micción, dolor en el flanco discreto y sin irradiaciones, infecciones de vías urinarias de repetición, sensación de vaciamiento incompleto de la vejiga, disminución en el calibre del chorro de la orina y pujo. En 3 ocasiones ha requerido incluso sondeo vesical por retención aguda de orina. En su examen físico el paciente se encuentra con signos vitales en rangos normales, abdomen blando, depresible, con hiperestesia en ambos flancos, de predominio izquierdo, próstata agrandada, de temperatura normal, lisa por examen rectal. Cual seria su posibilidad diagnóstica mas importante

a) b) c) d)

Pielonefritis Litiasis renoureteral Hipertrofia prostática benigna Sx. Nefrótico

Síntomas.

No se ha encontrado una relación directa entre la severidad de los síntomas y el tamaño de la próstata. Algunos hombres presentan síntomas severos de obstrucción urinaria pero con un crecimiento prostático mínimo, mientras que otros presentan síntomas mínimo pero próstatas grandes. La hipertrofia del músculo detrusor de la vejiga puede compensar inicialmente los síntomas de obstrucción urinaria. Algunos de los síntomas de obstrucción urinaria son: 􀀺 Pujo al iniciar el vaciado; 􀀺 Flujo urinario débil y/o un vaciado prolongado; 􀀺 Goteo terminal; 􀀺 Sensación de vaciado incompleto; 􀀺 Nicturia; 􀀺 Incontinencia y 􀀺 Retención urinaria aguda. Los síntomas irritativos incluyen: 􀀺 Disuria; 􀀺 Frecuencia; 􀀺 Urgencia. Estos síntomas pueden verse exacerbados por medicamentos para la gripe, antihistamínicos sedantes, aquellos con efectos antimuscarínicos como los antidepresivos tricíclicos, la ingesta de alcohol y la inmovilización. El puntaje internacional de síntomas prostáticos (IPSS) permite evaluar los síntomas de manera objetiva y reproducible según sean leves, moderados o graves.

GUIA DE PRACTICA CLINICA HIPERPLASIA PROSTATICA BENIGNA Diagnóstico y Tratamiento P. Bibliografía Guías Urologicas Hiperplasia Prostática Benigna 1. Clinical Practice Guidelines , Number 8. “ Benign Prostatic Hyperplasia: Diagnosis and Treatment. Agency for Health Care Policy and Research. AHCPR Publication No. 94-0582 February 1994. 1.A. Proceedings – 4Th. International Consultation on Benign Prostatic Hyperplasia ( BPH), Paris July 2-5,1997, Denis L., Griffiths K., Khoury S., Cockett A.T.K., Mc Conell., Chatelain C., Murphy G., Yoshida O. 2. Sagnier P P, Macfarlane G, Richard F et al. Impact of symptoms of prostatism on bothersomeness and quality of life of men in the French community. J Urol 1995; 153:669-673. 3. Guess H A . Prevalence of BPH in community surveys. IN: Garraway W M ( ed) The epidemiology of prostate disease. Heidelberg: Springer-Verlag, 1995:121-131.

44.- Un hombre de 25 años de edad refiere una masa testicular no dolorosa, de consistencia dura y de 3cm de tamaño, la cual descubrió mientras se bañaba. El EF confirma que la masa viene del testículo, no es parte del epidídimo y es sólida. El resto de EF normal. ¿Cuál es el siguiente paso?

a) b) c) d)

BAAF trans-escrotal de la masa Biopsia incisional trans-escrotal Orquiectomía trans-escrotal Orquiectomía radical inguinal

TRATAMIENTO Ante la presencia de una masa testicular sospechosa (aumento de volumen, consistencia o irregularidad testicular, seguido de ecografía testicular compatible con probable tumor), se debe actuar de la siguiente manera:

1) Determinación de marcadores tumorale serológicos. 2) Orquiectomía por vía inguinal previo clampeo de cordón espermático. 3) Nunca se debe realizar abordaje por vía escrotal. 4) Nunca se deben practicar «biopsias» testiculares. 5) Una vez realizado el diagnóstico histológico, se efectuarán los estudios por imágenes (radiografía de tórax y tomografía de abdomen y pelvis), nueva determinación de marcadores tumorales y laboratorio necesarios para evaluar la extensión de la enfermedad y de esta manera, asignarle una categoría pronóstica. Para hacer más sencilla la comprensión vamos a describir los distintos tratamientos según el estadio (I – II – III) y tipo celular (seminoma o no seminoma). Cáncer testicular en estadio I Seminoma — El seminoma en estadio I tiene una tasa de curación de más de 95%.(fig. 1). Opciones de tratamiento: 1) Orquiectomía inguinal radical seguida de radioterapia a los ganglios linfáticos inguinales y retroperitoneales ipsilaterales. Sólo se requieren dosis bajas de radiación. Los ganglios retroperitoneales se irradian profilácticamente debido a que aproximadamente el 15% presenta propagación ganglionar oculta. Los tumores de mayor tamaño, con invasión vascular o de rete testis, parecerían tener mayor riesgo de metástasis ganglionares. 2) La otra opción es la orquiectomía inguinal radical sin irradiación de ganglios retroperitoneales seguida por la determinación frecuente de marcadores séricos, radiografías de tórax y tomografías computadas (vigilancia). De esta manera, serán tratados con radio o quimioterapia, sólo los pacientes que recaen. Con ambas conductas se obtienen cifras similares de curación. TRATAMIENTO DEL CÁNCER DE TESTÍCULO Dres. Carlos A. Delfino, Graciela M. Caccia, Jorge L. Hidalgo

45.- Masculino de 14 años que presenta percepción odorífera desagradable sin causa aparente, posteriormente se agrega movimiento tónico –clónico generalizado de 2 minutos de duración quedándose dormido posteriormente. Usted piensa que el paciente presentó:

a) b) c) d)

Crisis Generalizada Crisis parcial simple secundariamente compleja secundariamente generalizada Crisis parcial compleja secundariamente generalizada Crisis parcial simple secundariamente generalizada

1. Crisis parciales A. Crisis parciales simples B. Crisis parciales complejas C. Crisis parciales secundariamente generalizadas 2. Crisis generalizadas (convulsivas o no convulsivas) A. Crisis de ausencia B. Crisis convulsivas tónicas, clónicas C. Tónicas D. Clónicas E. Mioclónicas F. Crisis atónicas 3. Crisis epilépticas no clasificadas Las crisis parciales complejas son aquellas que se originan en áreas corticales no específicas, presentan alteración de la conciencia y cursan con descargas eléctricas en las regiones temporales o fronto-temporales. Son las más frecuentes en la consulta diaria y son de difícil control. Clínicamente se pueden manifestar exclusivamente con alteración de la conciencia por lo que pueden confundirse con las ausencias típicas llamadas antes "pequeño mal" pero clínicamente diferenciables. Las crisis parciales complejas se pueden iniciar como una crisis parcial simple seguida de un deterioro de la conciencia. Con frecuencia el paciente manifiesta una sensación somática generalmente molesta descrita como un trastorno epigástrico que asciende hacia la garganta y puede asociarse con sensaciones raras en la boca y los labios y tener deglución involuntaria. En otras ocasiones hay sensaciones de irrealidad como despersonalización; otras veces el paciente tiene trastornos de memoria (sensación de lo ya visto, de lo ya vivido) o bien alteraciones afectivas como son episodios de ansiedad, miedo, depresión o sentimientos paranoides. Estas alteraciones habitualmente son seguidas de una desconexión del medio ambiente y aparecer luego automatismos primarios como por ejemplo: escupir, frotarse la ropa, tratar de desvestirse, caminar, etc. Cuadro 2. Diferencias entre crisis de ausencia

y la epilepsia parcial compleja Características

Ausencias

Crisis parciales complejas

Edades

4-12

Cualquier edad

Causa

Idiopática

Secundaria a traumatismo, anoxia, infección

Frecuencia

Numerosas

Menos frecuentes

Duración

Segundos

1-3 minutos

Síntomas preictales

Ninguno

Siempre presentes

Síntomas postictales Ninguno

Somnolencia, cefalea, confusión

Complejos espiga-onda Alteraciones focales en un lóbulo 3CPS temporal

E.E.G.

BIBLIOGRAFIA 1. 2. 3.

4.

5.

6.

7.

Wyllie E (Ed).The Treatment of Epilepsy: Principles and Practice. 2nd ed. Baltimore, Williams and Wilkins, 1996 Roger J, Bureau M., Dravet C, et al. Epileptic syndromes in infancy, chilhood and adolescence. London: John Libbey, 1985. Commision on classification and terminology of the International League Against Epilepsy. Proposal for revised classification of epilepsies and epileptic syndromes. Epilepsia 1989; 30:389-399. Commision on classification and terminology of the International League Against Epilepsy. Proposal for revised clinical and electroencefalographic classification of epileptic seizures. Epilepsia 1981; 22:489-501. Commision on classification and terminology of the International League Against Epilepsy. Proposal for classification of epilepsy and epileptic syndromes. Epilepsia 1985; 26:268-278. Delgado-Escueta AV, Serratosa JM, Liu A, Weissbecker K, Medina MT, Gee M, Treiman LJ, Sparkes R. Progress in Mapping Human Epilepsy Genes, Epilepsia 35 (Suppl,1)S29-S40,1994 Delgado-Escueta AV, Serratosa JM and Medina MT. Juvenile Myoclonic Epilepsy:484-501.

46.- Varón de 31 años de edad que desde hace tiempo tiene episodios de ojo rojo con ausencia de secreción. Refiere leve sensación de cuerpo extraño y una carnosidad que crece y ya alcanzó la córnea. A la exploración se observa un tejido de neoformación nasal que invade 1mm de la córnea.

a) Carcinoma intraepitelial de conjuntiva b) Pingüecula c) Nevo conjuntival

d) Pterigion

El pterigión constituye una hiperplasia fibrovascular de carácter benigno de la conjuntiva bulbar que invade la córnea,1-4 está clasificado dentro de las degeneraciones no involutivas o tumoraciones epiteliales benignas corneales.1,3,5 Se localiza en la conjuntiva bulbar cerca del limbo corneal en el área interpalpebral, a las 3 y 9 horas. Puede ser unipolar (solo afecta una parte) o bipolar, cuando afecta tanto la parte temporal como la nasal, es más frecuente en el lado nasal. También pueden ser unilaterales o bilaterales. Recibe su nombre por su aspecto de ala pequeña. Presenta una forma triangular, cuyo vértice mira hacia el área pupilar y cuya base se orienta hacia la carúncula en la localización nasal, y hacia el lado temporal en los localizados en el área temporal. Habitualmente muestra un crecimiento horizontal que puede llegar a afectar el eje visual.2,3,7

Actualmente, en los estudios sobre alteraciones ultraestructurales del pterigión, se considera una alteración inflamatoria y proliferativa de la superficie ocular.2 Se plantea que las células madre limbares se modifican con exposición crónica a la luz ultravioleta, por lo que hay una rotura de la barrera limbar que causa la invasión conjuntival del epitelio corneal. Otro mecanismo que se expone relacionado con las reacciones inmunológicas es una reacción de hipersensibilidad tipo I a elementos irritantes exógenos (polvo, viento, etc.) asociada a una inflamación local que causa un incremento en la producción de IgE.3 Un estudio reciente muestra muestra una asociación entre una película lagrimal inestable y el inicio de un pterigión. Referencias bibliográficas 1. Adamis AP, Stark T, Kenyon KR. The manegement of pterygium. Ophtamol Clin North Am. 1990;3(4):611 2. Klinworth GK. Chronic Actinic keratopathy, a condition associated with conjunctival elastosis (pingueculae) and typified by characteristic extracellular concretions. Am J Pathol. 1972;67:32. 3. Mac Kenzie FD, Hirst LW, Battistutta D. Risk analysis in the development of pterygia. Ophthalmology. 1992;99:1056-61. 4. Clear AS, Chirambo MC, Hutt MSR. Solar keratosis, ptert-gium, and squamous cell carcinoma of the conjunctiva in Malawi. Br J Ophthalmol. 1979;63:102-9. 5. Sánchez Thorin JC, Rocha G, Yelin 313. Meta-analysis on the recurrence rales after bare sclera resection with and without mitomycin C use and conjunctival autograft placement in surgery for primary pterygium. Br J Ophthalmol. 1998;82:661-5

6. Lam DSC, Wong AKK, Fan DSP, et al. Intraoperative mito-mycin C to prevent recurrence of pterygium after excision: a 30-month follow-up study. Ophthalmology. 1998;105:901-5. 7. Tan DTR, Lim ASM, Goh RS, Smith DR. Abnormal ex-pression of the p53 tumor suppressor gene in the conjunc-tiva of patients with pterygium. Am J Ophthalmol. 1997;123:404-5.

47.- Se trata de paciente de 17 años de edad que es enviado para valoración por ortopedia, el paciente refiere dolor cervical bien delimitado el cual no mejora con AINES, en la radiografía se aprecia una imagen radiolúcida y expansiva de 4 cm de diámetro en el pedículo de la vértebra T12. ¿Cuál de las siguientes lesiones tumorales será la más probable?:

a) Encondroma. c) Tumor de Ewing. b) Osteoblastoma. d) Metástasis de cáncer de pulmón.

Osteoteoblastoma Benigno Llamado también osteoide gigante; conformado por células de naturaleza osteoblástica, formador de tejido óseo y osteoide, bastante vascular, generalmente de más de 2 cm de diámetro; carece de dolor típico y de hueso reactivo y tiene un cierto potencial de crecimiento. No es frecuente; más en el sexo masculino, entre 10 y 25 años; evidente preferencia por la columna vertebral, incluyendo el sacro, fémur, tibia, huesos tubulares cortos de manos y pies. • Clínica El dolor, generalmente local, es el síntoma cardinal, sin tener la intensidad típica de las algias del osteoma osteoide; es producido por la compresión por el tumor, ya sea de médula o nervios radiculares y ocasiona a veces paraparesia o paraplejia, otras veces escoliosis, espasmo muscular y síntomas neurológicos. • A RX Zona lítica,radiolucida de 2 a 10 cm de diámetro rodeada por capa de osteoesclerosis densa, hueso expandido y engrosado; cuando están localizados en tejido esponjoso hay ausencia habitual de osteoesclerosis perifocal. • Anatomía patológica Macroscópica: bien limitado, hemorrágico, granuloso y friable; el componente osteoide es de grado de calcificación variable; el hueso adyacente no muestra osteoesclerosis, salvo en los huesos diafisarios, en que hay zona de hiperostosis. Microscópica: Los osteoblastos con núcleos regulares poco cromatínicos y con abundante protoplasma producen trabéculas entrelazadas o discretos islotes de sustancia osteoide o tejido óseo; hay amplia variación en la tipología microscópica, produciendo confusión con los tumores a células gigantes, osteomas osteoides, osteoma y quiste óseo aneurismático; hay reabsorción osteoclástica y también hueso más maduro en reconstrucción; nunca se observa

formación de cartílago; no hay polimorfismo celular, aunque en lesiones jóvenes a veces se observan figuras mitóticas que pueden confundir con sarcomas. Al tener una histología similar al osteoma osteoide, nadie ha observado que un osteoma osteoide crezca o que un osteoblastoma haya tenido de inicio un osteoma osteoide. A veces maligniza. • Tratamiento El de elección es curetaje de toda la lesión, seguido de colocación de injertos si fuese necesario; si la localización lo permite, la escisión en bloque es aconsejable. No es aconsejable la radioterapia.

Tumores Óseos en general. Tumores Benignos de los Huesos Dr. Luis Julio Huaroto Rosa-Pérez Cirugía: II cirugía ortopédica y traumatología Autor: Universidad Nacional Mayor de San Marcos (Lima). Facultad de Medicina. Escuela Académico Profesional de Medicina Humana. Departamento Académico de Cirugía Publicación: Lima: UNMSM, 2000 Descripción: 407 p. : il., tablas, fotos ; 24 cm.

Serie: (Cirugía; 2) ISBN: 9972-46-102-5 Otros autores: Salaverry García, Oswaldo, 1959-, ed. Tema: Traumatología; Ortopedia

48.- Se trata de paciente masculino de 33 años el cual presenta los siguientes datos clínicos, álgia facial, rinorrea posterior , obstrucción nasal, éstos son criterios mayores para el diagnóstico de:

a) b) c) d)

Sinusitis aguda Faringoamigdalitis estreptococica Rinitis vasomotora Rinitis estacional

CRITERIOS CLÍNICOS El diagnostico de la sinusitis aguda es clínica, depende en la presencia de por lo menos dos síntomas mayores, o un síntoma mayor y dos menores. Síntomas mayores:    

Dolor o presión facial. Obstrucción nasal. Rinorrea purulenta. Hiposmia o anosmia.

Síntomas menores:     

Cefalea. Halitosis. Dolor dental superior. Tos, especialmente en niños. Otalgia o presión en oídos.

La combinación de tres de cada cuatro criterios ha dado una especificidad del 81% y una sensibilidad del 66%. En estudios realizados por Williams y cols. encontraron en hombres adultos, que la congestión nasal, tos, estornudos tienen sensibilidad del 70% al 72%. El síntoma más específico (93%) es el dolor en senos maxilares. En un estudio en Europa, el indicador más sensible fue el dolor en los dientes (83%). El estándar de oro para el diagnóstico de la sinusitis maxilar es el hallazgo de material purulento a través de la aspiración del seno maxilar. Williams Jr JW, Aguilar C, Cornell J, Chiquette E. Dolor RJ, Makela M, Holleman DR, Simel DL. Antibiotics for acute maxillary sinusitis. Cochrane Database of Systematic Reviews 2003, Issue

49.- Un estudiante es tacleado mientras jugaba football y desarrollo dolor en rodilla severo. A la EF la rodilla se encuentra edematizada y el paciente tiene dolor a la palpación directa del aspecto lateral de la rodilla. Cuando se flexiona la rodilla 30°, la aducción pasiva ocasiona dolor en la misma área, y la pierna puede ser aducida más que la pierna contralateral. Cajón anterior, cajón posterior y Lachman negativos. ¿Cuál es el sitio más probable de lesión?

a) b) c) d)

Ligamento cruzado anterior Ligamento colateral lateral Menisco lateral Ligamento cruzado posterior

Se produce por un trauma en varo al golpearle en la parte externa de la pierna por debajo de la rodilla o al caerle otro jugador o contusión en la cara interna de la rodilla. Presenta dolor y generalmente sensación de parestesia en el territorio del nervio ciático poplíteo externo por distensión, que puede ser una parestesia transitoria en traumas de menor energía o una lesión definitiva con pie caído en traumas de alta energía. El paciente presenta dolor, derrame leve a moderado y equimosis en cara lateral de la rodilla. El diagnostico se hace al encontrar: 1. Bostezo en varo en 30 grados de flexión: para evaluar el ligamento colateral lateral, el dolor es en trayecto del ligamento. Puede ser de diferente magnitud: 

Grado I: muy discreta apertura de la interlinea comparado con lado contra lateral.



Grado II: apertura de 5 mm de la interlinea articular.



Grado III: apertura de 10 mm de la interlinea articular.

2. Bostezo en varo en extensión completa: para evaluar esquina postero lateral, capsula postero lateral y ligamento fibulo poplíteo. Las estructuras laterales tienen un potencial de cicatrización menor que las estructuras mediales. En las lesiones grado I y II se debe inmovilizar y proteger el apoyo durante seis semanas. Se debe tener un alto índice de sospecha y en los casos grado III y donde se presente compromiso de la capsula postero lateral, se debe realizar cirugía para reparo primario en las primeras dos semanas, pues el reparo primario funciona mejor que cualquier reconstrucción anatómica

50.- Una mujer de 55 años, se queja de presión pélvica y una masa en la entrada vaginal, hace 3 años tuvo su último periodo menstrual. No tomo terapia de reemplazo, tiene dificultad para evacuar. Tiene una tos crónica y una historia de tabaquismo positivo a razón de 30 cajetillas año. Ha tenido 3 partos y el ultimo bebe pesó 4,500 gramos. Tiene una orina de 60 cc. ¿Cuál de los siguientes hallazgos es más probable que encuentres en la exploración pélvica? a) b) c) d)

Rectocele Cistocele Enterocele Uretrocele

El Rectocele es una hernia de la pared anterior del Recto hacia la porción posterior de la vagina. La incidencia real de esta “deficiencia anatómica” es desconocida y en muchas ocasiones es un resultado del paso del tiempo. Es un hallazgo muy frecuente del examen perineal, siendo en múltiples ocasiones asintomático. El rectocele puede ser un hallazgo importante del síndrome de Obstrucción Defecatoria (SOD). No debe tomarse como una deficiencia anatómica única, sino como parte importante de un problema anatomo-fisiológico complejo. Un principio importante es la etiología y anatomo-patología del rectocele. Existen varias teorías sin consenso principal. La existencia, deficiencia o alteraciones del septo rectovaginal son controversias importantes. No existe una fascia visceral que separe el recto de la vagina o que forme un septo específico. Existe frontera entre donde termina la pared anterior del recto y donde comienza la pared de la vagina, pero múltiples estudios no han encontrado un septo rectovaginal específico. El septo puede estar formado de una película casi transparente hasta una pared de consistencia fibromuscular fuerte. El rectocele es un hallazgo común. Se presenta en el 80% de las pacientes femeninas y 13% de los masculinos en una defeco grafía (> a 1cm.) Entre mas grande es el rectocele, mayor son los síntomas asociados, Dificultad en la evacuación, constipación crónica, dolor rectal y perineal, sensación de masa y en ocasiones sangrado. La necesidad de presión manual para ayudar a la evacuación o para vaciar el rectocele es común en más del 50% de los pacientes. El diagnostico se hace con un simple tacto rectal e inspección vaginal, pero debe recordarse que rara vez es el rectocele un hallazgo aislado. La presencia cistocele, peritoneocele, enterocele u otros prolapsos perineales y problemas funcionales deben ser descartados antes de proponer la reparación quirúrgica del rectocele. Defeco grafía, Pruebas de función fisiológica del piso pélvico, pruebas de función urinaria, evaluación del esfínter anorectal por ultrasonido y hasta la resonancia magnética han sido propuestos antes de la cirugía.

Bibliografía: Rectocele: Pathogenesis and surgical managment. Zbar AP, Linemann A, Fritsch H, Beer-Gabel M, Pescatori M. Int J Colorectal Dis. (2003) 18:369-384. Evaluation and Treatment of Women with rectocele. Cundiff GW, Fenner D, Obstetrics and Ginecology 104(6): 1403-1416 Stapled transanal rectal resection to treat obstructed defecation caused by rectal intussusseption and rectocele. Renzi A, Izzo D, Di Sanrno (2—6) 21:661-667 Rectocele repair using biomaterial augmentation. Altman D, Melgren A, Zetterstrom J.

Obstet Gynecol (2005) 60(11)753-760.

51.- Masculino de 2 años que acude al servicio de consulta externa por presentar otorrea bilateral mucopurulenta de una semana de evolución, como antecedentes ha presentado cuadros repetitivos de infecciones de vías respiratorias altas en todos ellos ha recibido diferentes tratamientos. A la exploración física presenta ambas membranas timpánicas opacas e íntegras a la rinoscopía presenta mucosa pálida y violácea con moco hialino en forma abundante, orofaringe sin alteraciones, cuello sin adenomegalias, la radiografía lateral de cuello muestra datos francos de: a) b) c) d)

Otitis media bilateral Mastoiditis bilateral Otomastoiditis bilateral Timpanitis bulosa

[1] La mayoría de los episodios de OMA (70-90%) coinciden con una infección de vías respiratorias altas, por lo que los síntomas de la misma suelen estar presentes, pero presentan escaso valor discrimitativo: Rinitis. Tos. Irritabilidad, llanto. Fiebre. Rechazo de la alimentación. [2] Uno o más de los síntomas siguientes elevan sustancialmente la probabilidad de otitis media aguda y, por lo tanto, hace preciso realizar un exámen otoscópico para corroborarlo:

Otorrea. Otalgia, tocarse la oreja. Falta de descanso nocturno, despertar frecuente durante la noche. La asociación otalgia y falta de descanso nocturno diagnostica el 71% (menores de 2 años) y 75% (mayores de 2 años) de los episodios de OMA. En menores de 2 años, la asociación de otalgia y conjuntivitis incrementa la probabilidad hasta el 76% y sugiere etiología por Haemophilus influenzae. Otros síntomas que en los libros clásicos se relacionan tradicionalmente con los episodios de otitis no aportan información adicional para el diagnóstico de OMA, dado que se presentan con la misma frecuencia en niños con otros cuadros: Fiebre. Vómitos, molestias abdominales. Diarrea. Hipoacusia. Vértigo.

[3] En el exámen otoscópico es necesario valorar la coloración, transparencia y movilidad del tímpano, siendo altamente sugestivos de otitis media aguda los siguientes hallazgos: Opacificación. Abombamiento. Disminución/ausencia de movilidad. En ausencia de sintomatología, la disminución de movilidad y opacificación son indicativos de OME. El enrojecimiento como signo aislado resulta muy poco específico. [4] Se considera afectación del estado general: Presencia de otalgia moderada/severa (irritabilidad, tocarse la oreja en 39ºC (oral) o 39.5ºC (rectal). También quedarían incluidos en este grupo, a efectos del tratamiento a seguir, aquellos niños cuyo seguimiento y control no pueda garantizarse. [5] Se consideran factores de riesgo que incrementan la posibilidad de infección por neumococo: Asistencia a guardería. Tratamiento antibiótico en el mes previo a la consulta.

52.- Se trata de paciente masculino con antecedentes de alcoholismo crónico es llevado a urgencias con un cuadro de desorientación, dificultad para recordar hechos de los días anteriores, errores en los razonamientos, marcha torpe y una desviación en los ojos divergente que antes no tenía. Refieren los familiares, ha estado bebiendo alcohol hasta unas horas antes de su arribo. El diagnóstico más probable es:

a) Alucinosis alcohólica.

b) Intoxicación etílica aguda c) Enfermedad de Korsakoff. d) Encefalopatía de Wemicke.

La encefalopatía de Wernicke es una enfermedad prevenible que es desencadenada por una deficiencia severa de vitamina B1 o tiamina. Se presenta comúnmente en personas con alcoholismo crónico, aunque también puede aparecer en personas con desnutrición secundaria a hiperemesis, diálisis renal, cáncer o hasta SIDA.

Fisiopatología La tiamina es un cofactor para diferentes enzimas como la transcetolasa, la piruvato deshidrogenasa o la alfacetoglutarato deshidrogenasa. Al bajar los niveles de tiamina, se reduce significativamente la capacidad metabólica del cerebro sobre la glucosa, originando daño mitocondrial. Al mismo tiempo que no hay un adecuado funcionamiento de la alfacetoglutarato deshidrogenasa, hay un acúmulo de glutamato, que al combinarse con una deficiencia importante de energía, provoca daño celular excitotóxico.

Manifestaciones clínicas La triada característica en la clínica de esta enfermedad es oftalmoplejía, ataxia y confusión global (aunque realmente solo 1/3 del total de los pacientes con encefalopatía de Wernicke lo presenten). La mayoría de los pacientes están desorientados, indiferentes e inatentos. También presenta alteraciones en el nervio oculomotor, incluyendo nistagmo horizontal en la vista lateral, parálisis del recto lateral normalmente bilateral, parálisis de la mirada conjugada y en menor frecuencia, ptosis. La ataxia de la marcha probablemente proviene de la combinación de una polineuropatía, involucro cerebeloso y paresia vestibular. Las pupilas pueden o no presentar miosis.

Se cree que la encefalopatía de Wernicke y el síndrome de Korsakoff son dos etapas de la misma afección. La encefalopatía de Wernicke es causada por cambios dañinos en el cerebro, generalmente debido a la falta de vitamina B-1 (tiamina), común en personas que sufren de alcoholismo. El consumo excesivo de alcohol interfiere con el metabolismo de la tiamina en el cuerpo. Incluso, si alguien que bebe alcohol en exceso consume una dieta bien balanceada, la mayor parte de la tiamina no es absorbida. El síndrome o psicosis de Korsakoff tiende a desarrollarse a medida que desaparecen los síntomas del síndrome de Wernicke. Ésta implica daño a áreas del cerebro involucradas en la memoria.

Enfermedades neurologicas relacionadas con el alcoholismo. Rev Clin Esp. 1998 Apr;198(4):226-33. de , Rubio R. Encefalopatia de Wernicke-Korsakoff secundario a hiperemesis no gravidica. Neurologia. 2009 Jan-Feb;24(1):75-7. Garcia C., Estela H., Ribera

53.- Acude a consulta paciente de 36 años de edad con los siguientes resultados de laboratorio encuentra glucosa de 172 mg/dl, una CPK de 429 U/L, GTP 62 U/L, GOT 43 U/L y GGT 26U/L. En el electrocardiograma presenta un bloqueo A-V de primer grado. En la exploración física se aprecian unas opacidades corneales incipientes y una dificultad en relajar un músculo después de una contracción intensa, siendo muy evidente en las manos. ¿Qué diagnóstico es el más probable?:

a) Una distrofia muscular de Duchenne. b) Una distrofia muscular de Steinert. c) Una distrofia muscular de Becker. d) Una distrofia muscular de cinturas. La distrofia miotónica o enfermedad de Steinert es la distrofia muscular más común en adultos y la segunda distrofia muscular más frecuente después de la distrofia muscular de Duchenne. Es un trastorno genético autonómico dominante que afecta a uno de cada 8000 individuos. El debut es usualmente en la segunda o tercera década y la esperanza de vida es de seis décadas. Se caracteriza por debilidad y atrofia de los músculos voluntarios de los ojos, la cara, el cuello, brazos y piernas, miotonía, cataratas, posteriores subcapsulares, defectos en la conducción nerviosa, trastornos endocrinos, déficit cognitivo y calvicie frontal. Los músculos relacionados con las actividades involuntarias como deglutir y respirar, así como los que rodean los órganos internos como el tracto digestivo alto y bajo, la vejiga urinaria y el útero, pueden ser afectados también cuando progrese la enfermedad en el individuo.

Bibliografía 1. Ropper AH; Brown RH. The muscular distrophies. En: Ropper AH;Brown RH. Adams and Victor´s Principles of Neurology. 8va ed. New York. Editorial McGraw – Hell;Interamericana; 2005. p. 1213-1229. 2. Atrofias musculares. En: Roca R; Smith V et al. Temas de Medicina Interna. 4ta Edición. La Habana. Editorial Ciencias Médicas; 2002 .p. 466-470. 3. Seznec H et al. Mice transgenic for the human myotonic dystrophy region with expanded CTG repeats display muscular and brain abnormalities. Hum Mol Genet. 2001 Nov 1; 10(23):2717-26. 4. Weinberg B, Bosma JF, Shanks JC, et al. Myotonic dystrophy initially manifested by speech disability. J Speech Hear Disord 1968; 33:51–9. 5. .Journal of Neurology Neurosurgery and Psychiatry 2004;75:1480-1482 6. Pelargonio G, Dello Russo A, Sanna T et al — Myotonic dystrophy and the heart. Heart, 2002; 88:665-670.

54.- Masculino de 33 años presenta episodios de cefalea periocular derecha, los cuales le despiertan por la noche, muy intensos, con mas de media hora de duración acompañado de lagrimeo, así como congestión nasal. Le hacen levantarse de la cama. ¿Cuál sería su diagnostico más probable?:

a) Cefalea en racimos. b) Neuralgia del trigémino. c) Migraña basilar. d) Sospecharía un tumor cerebral o una hipertensión intracraneal

La cefalea en racimos (CR) es una de las cefaleas más terriblemente invalidantes. Desde el punto de vista clínico los pacientes que la sufren describen un dolor atroz, lancinante referido a la región periocular y que se acompaña de una serie de síntomas y signos, fundamentalmente de disfunción autonómica, como lagrimeo, edema palpebral o rinorrea, por citar sólo algunos, que facilitan su diagnóstico. Sin embargo, a pesar de su característico perfil clínico continúa siendo pobremente reconocida y mal controlada. Más aún, a pesar de los avances existentes en el tratamiento de este tipo de cefalea, muchos de los pacientes, a pesar del sufrimiento que les comportan sus crisis, reciben tratamientos inespecíficos e ineficaces. Criterios Diagnósticos

Tratamiento Los enfermos son tratados habitualmente con analgésicos u otros fármacos destinados a la migraña común, enfermedad con la que no tiene mucha relación, razón por la cual los tratamientos tienen poco efecto. Los tratamientos se dividen en abortivos que alivian un ataque individual y preventivo, que intentan combatir un ciclo completo. Entre los abortivos se puede mencionar al oxígeno puro a un ritmo de diez-doce litros por minuto, las ergotaminas y los triptanos (sumatriptán y zolmitriptán). Estos últimos registran la mayor tasa de eficacia aunque su alto precio y sus efectos vasoconstrictores son factores limitativo de su prescripción. Se ha descrito el uso de prednisona un corticoide como preventivo. En algunos países, como Francia, la sanidad pública subvenciona bajo determinadas condiciones la totalidad del precio de estos analgésicos. En tratamiento de urgencia, al enfermo se le hace inhalar oxígeno, lo que suele tener un efecto también analgésico. Sin embargo, es frecuente que los facultativos de urgencias no sean capaces de detectar la enfermedad, dado que su incidencia es escasa, y traten al enfermo con analgésicos y ansiolíticos. Por este motivo, existen iniciativas de asociaciones de afectados tendentes a crear un carné de enfermo, validado por las autoridades sanitarias, que describan el tratamiento a aplicar en caso de ataque. Existe controversia sobre la posible eficacia de una serie de tratamientos herbales (Kudzu) y hormonales (Melatonina). En casos de cluster refractario, con necesidad de múltiples tratamientos preventivos y sintomáticos, puede estar indicada la cirugía funcional mediante estimulación cerebral profunda (Deep Brain Stimulation - DBS) de núcleos hipotalámicos o bien mediante estimulación del nervio suboccipital. 1.

Sewell, R. Andrew, M.D.; Halpern, John M., M.D. "The Effects Of Psilocybin And LSD On Cluster Headache: A Series Of 53 Cases." Abstract. Presented to the National Headache Foundation’s Annual Headache Research Summit. February, 2006.

55.- Se trata de paciente masculino de 37 años de edad, al caer de su motoclicleta sufre traumatismo craneal llega consciente al Servicio de Urgencias. Al realizar radiografías se aprecia una fractura lineal de la bóveda craneal. A LAS 12 horas del accidente comienza a reducirse de forma progresiva el nivel de conciencia, observándose asimetría pupilar. ¿Qué diagnóstico de los siguientes debe hacerse en primer lugar?: a) Una crisis epiléptica postraumática b) Una meningitis c) Un hematoma epidural d) Un coma metabólico yatrogénico.

El hematoma epidural intracraneal es una hemorragia venosa o arterial que se situa entre el cráneo y la duramadre, complicación que puede ocurrir después de traumatismos craneoencefálicos (TCE) aparentemente banales. Su identificación y evacuación quirúrgica

precoz es muy importante ya que puede dar lugar de forma brusca,tras un intervalo lúcido variable, a una compresión cerebral y herniación. Etiología Ocurre en aproximadamente el 1% de todos los TCE que ingresan y en el 22% de los casos con fractura de cráneo. Es cuatro veces más frecuente en el sexo masculino. Ocurre generalmente en adultos jóvenes, y es raro antes de la edad de 2 años o después de los 60 (quizás porque la dura es más adherente a la tabla interna en estos grupos). Más raro aún es la presentación en recién nacido que se da con mayor frecuencia en madres nulíparas (2). La localización más frecuente es a nivel temporal 60% con epicentro sobre pterion. La localización frontal, parietal, occipital y fosa posterior se produce en 5-10% de los casos cada uno de ellos.

Clínica La pérdida breve de conciencia seguida de un intervalo lúcido (que dura desde unos cuantos minutos hasta horas) con posterior pérdida de conciencia, hemiparesia y dilatación de la pupila ipsilateral se presenta con una frecuencia de < 10 a 27 %. El 60 % no presentan pérdida de conciencia inicial. Este tipo de hematoma es más frecuente en adultos jóvenes y puede desarrollarse en ausencia de fractura de cráneo, especialmente en niños e individuos jóvenes, pues en estos el cráneo es más elástico que en los adultos. Otros síntomas y signos serían los siguientes: Midriasis del lado del hematoma (60 % de los casos) por compresión del tercer par craneal (signo de compresión de la región superior del meséncefalo, que pueden consistir en hemiparesia o rigidez descerebración y por último datos de afección del tallo encefálico y muerte). Al final se desarrolla una hernia transtentorial. Tumefacción en piel cabelluda de región temporo-parietal. Otorrea homolateral al sitio del hematoma Cefalea unilateral de las lesiones Inquietud y vómitos, principalmente en niños Alteraciones sensitivas, hemihipoestesia, afasia hemianopsia homónima. Posturas especiales, giro de cabeza, desviación conjugada de mirada hacia el lado de lesión. Ataque de convulsiones generalizadas

Signos de parálisis del tercer par craneal, ptosis palpebral, midriasis y estrabismo divergente. Los hematomas epidurales pueden clasificarse según la aparición de hipertensión endocraneana: 1. Agudo: Cuando la hipertensión endocraneana es en las primeras 48 horas post trauma.(lo más frecuente) 2. Subagudo: cuando los signos aparecen entre el segundo día y una semana postrauma. 3. Crónico: Los signos son visibles después de una semana; en ocasiones puede haber afectación del tercer y sexto par craneal dando parálisis completa del ojo afectado. Se puede agravar con un aumento agudo de presión endocraneana, los signos en este caso serían los siguientes: Respiración irregular Bradicardia Hipertensión ocasional

La hemiparesia ipsilateral se puede producir por compresión del pedúnculo cerebral opuesto en la incisura tentorial (fenómeno de Kernohan o de incisura tentorial). Kernohan y Woltman lo describieron en 1929. Clínicamente se traduce por una focalidad ipsilateral, y en apariencia incongruente, a la lesión original. Esta lesión anatomopatológica, denominada Hendidura de Kernohan (Kernohan's notch) debido a su apariencia de muesca o surco, está bien documentada mediante RM en un trabajo de J. Giménez-Pando en la revista Neurocirugia 2004 Aug;15(4):384-7, concluyendo que ante un paciente con focalidad neurológica ipsilateral a una lesión expansiva supratentorial en el cual la TC no explique la clínica, una RM podría demostrar lesión del pedúnculo cerebral contralateral. Diagnóstico En la RX de cráneo no se muestra la fractura en el 40 % de los casos. La mayoría son menores de 30 años. TAC cerebral y craneal

La lesión es biconvexa de alta densidad entre el cráneo y la masa encefálica, en el 84% de casos. En el 11% convexo y luego de distribución recta y en el 5 % se asemeja al hematoma subdural. Es generalmente uniforme en su densidad, bordes definidos, contiguo con la tabla interna. En raras ocasiones el hematoma epidural es isodenso y solamente se visualiza tras la inyección de contraste (3).

Si se han identificado fracturas craneales aunque el TAC cerebral sea normal, la presión intracraneal (PIC) sea normal se realizará TAC de control en los días siguientes para descartar un hematoma epidural tardío y por supuesto un TAC inmediato ante cualquier deterioro clínico-neurológico (5). Diagnóstico Diferencial Incluye una presentación posttraumática descrita por Denny-Brown consistente en un intervalo lúcido; seguido de bradicardia y vómitos. Los niños llegar a estar somnolientos y confusos. Teoría: una forma de síncope vagal, pero que debe ser diferenciada del hematoma epidural.

Bibliografía 1. 2.

3.

Agrawal D, Cochrane DD: Traumatic retroclival epidural hematoma-a pediatric entity? Childs Nerv Syst, 2006. Heyman R, Heckly A, Magagi J, Pladys P, Hamlat A: Intracranial epidural hematoma in newborn infants: clinical study of 15 cases. Neurosurgery 57:924-929; discussion 924-929, 2005. Mendonca R, Lima TT, Dini LI, Krebs CL: Bilateral isodense epidural hematoma: case report. Arq Neuropsiquiatr 63:862-863, 2005.

4.

5.

Paterakis KN, Karantanas AH, Hadjigeorgiou GM, Anagnostopoulos V, Karavelis A: Retroclival epidural hematoma secondary to a longitudinal clivus fracture. Clin Neurol Neurosurg 108:67-72, 2005. Radulovic D, Janosevic V, Rakic M, Durovic B, Slavik E, Lakicevic N: [Delayed epidural hematoma after mild head injury]. Vojnosanit Pregl 62:679-682, 2005.

56.- Masculino de 66 años que acude al servicio de urología, al realizar un PSA en sangre demuestra 20 ng/L. Se realiza ecografía la cual muestra un nódulo de 2 cms, localizado en el lóbulo izquierdo, en el que la biopsia demuestra carcinoma.Se realizan estudios de extensión manifiesto de metástasis a distancia, adenopatías sospechosas ni invasión de órganos vecinos. Se le realiza prostatectomía radical. ¿Cuál de los siguientes parámetros tiene importancia pronóstica?:

a) La estadificación anatómica en la pieza de resección y el volumen total de la glándula. b) La estadificación anatómica en la pieza de resección y el score de Gleason. c) La estadificación anatómica en la pieza de resección y el grado tumoral según BloomRichardson. d) El volumen tumoral estimado en comparación con el volumen glandular total.

Grado Gleason Los análisis univariantes y multivariantes de factores pronósticos en el cáncer de próstata identifican el índice de Gleason como uno de los marcadores pronósticos más significativos, con peores resultados de supervivencia, extensión tumoral y periodo libre de enfermedad cuanto más indiferenciado esté el tumor4. La utilización de índices Gleason combinados (que indican la proporción relativa de muestras con cáncer de alto grado) nos proporciona una información pronóstica más precisa. Si valoramos el índice Gleason junto con el estadio clínico haremos estimaciones pronósticas aún más acertadas35. Sin embargo, se ha encontrado que cuando el tumor es de alto grado, el pronóstico será desfavorable incluso cuando exista órgano-confinación. El índice Gleason más preciso se obtiene con la pieza de prostatectomía radical. Cuando se intenta estimarlo a partir de la muestra obtenida con biopsia por punción se comete un alto porcentaje de errores, superior incluso al 50%. Algunos estudios sugieren que el error más frecuente ocurre cuando la biopsia por punción sugiere un Gleason
View more...

Comments

Copyright ©2017 KUPDF Inc.
SUPPORT KUPDF